RE: [obm-l] Apertos de maos (logica)

2008-01-22 Por tôpico Qwert Smith

Sejam os casais:
A1, A2, B1, B2, C1, C2, D1, D2, E1, E2
 
O numero maximo de apertos de mao e 8
O numero minimo de apertos de mao e 0
 
Se Maria escutou 9 numeros diferentes entao:
 
1 - Como temos nove possibilidades, Maria apertou o mesmo numero de maos que 
outra pessoa
(9 numeros, 10 pessoas)
2 - Nenhum dos outros casais apertou o mesmo numero de maos
de 1 e 2, temos 3- Maria apertou o mesmo numero de maos que Joao
 
Alguem apertou 8 maos, seja essa pessoa A1.
4 - Sobram como possibilidades pra B12,C12,D12 e E12  numeros de 1 a 7 
inclusive, ja que todos apertaram a mao de A1.
de 4 vem que A2 apertou 0 maos.
 
Por raciocinio analogo temos que os casais se agrupam por numero de apertos 
como:
(0,8), (1,7), (2,6), (3,5) e (4,4)
 
Se Maria e Joao apertaram o mesmo numero de maos ambos apertaram 4 maos
 Date: Tue, 22 Jan 2008 12:11:24 -0200 From: [EMAIL PROTECTED] To: 
 obm-l@mat.puc-rio.br Subject: [obm-l] Apertos de maos (logica)  Ola' 
 pessoal, este e' simples, mas interessante:   Joao e Maria participaram 
 de uma festa onde havia mais 4 casais, num total de 10 pessoas.  Durante a 
 festa, houve varios apertos de maos, e, ao final, quando Maria perguntou 
 quantas maos diferentes cada um tinha apertado, ouviu - muito surpresa - 9 
 numeros diferentes entre si.  Sabendo-se que nenhum casal se cumprimenta 
 (isto e', Joao e Maria, por exemplo, nao se cumprimentam), e que todos os 
 cumprimentos sao feitos apenas com a mao direita, pergunto:  Quantas maos 
 diferentes Joao apertou?  []'s Rogerio Ponce 
 = 
 Instruções para entrar na lista, sair da lista e usar a lista em 
 http://www.mat.puc-rio.br/~obmlistas/obm-l.html 
 =
_
Need to know the score, the latest news, or you need your Hotmail®-get your 
fix.
http://www.msnmobilefix.com/Default.aspx

Re: [obm-l] Dragoes e passarinhos (Logica)

2007-08-15 Por tôpico Qwert Smith
Acho que fica mais facil de tras pra frente... Vamos supor que so C e D tem 
olhos verdes.
D so ve um dragao com olhos verdes, em 4 segundos ele chega a conclusao que 
ele e o segundo e vira passarinho.  Um segundo antes Claude teria virado 
passarinho.  Em 4 segundos, 2 passarinhos.  Os casos pra [A e B], [A e C], 
[A e D], [B e C] e [B e D] sao similares com tempo total de 2, 3, 4, 3 e 4 
segundos respectivamente.


Agora vamos supor B, C e D.  D ve que B e C tem olhos verdes.  Como apos 2 
segundos B ainda e dragao, D deduz que B tem que estar vendo 2 dragoes tb e 
logo, 4 segundos depois, D vira passarinho.  Ou seja D leva, nesse caso, 6 
segundo pra virar passarinho.  Mas e B e C? B e C levam ambos 5 segundos pra 
virarem passarinhos (porque?).  Para [A, C e D] os tempos de transformacao 
seriam 5 segundos pra D, 4 segundos pra [A e C]. Pra [A, B e D] seriam os 
mesmo 5 segundos pra D, e 3 segundos pra [A e B].  Para [A, B e C] seriam 4 
segundos pra C e 3 segundos pra [A e B]


Analisando para A, B, C e D vemos que A precisa esperar 5 segundos pra 
perceber que o caso em questao nao e [B, C e D]. A vira passarinho em 6 
segundos.  B precisa de 6 segundos. C precisa de 6 segundos. D precisa de 7 
segundos.  Eu acho... ficou meio confuso



From: Rogerio Ponce [EMAIL PROTECTED]
Reply-To: obm-l@mat.puc-rio.br
To: obm-l@mat.puc-rio.br
Subject: Re: [obm-l] Dragoes e passarinhos (Logica)
Date: Wed, 15 Aug 2007 07:46:24 -0300 (ART)

Ola' Ronaldo,
Sera' que Amadeus pode concluir que tem olhos verdes?
Ao irem para o ensolarado jardim, cada dragao certamente via a cor dos 
outros olhos.
Franz havia dito que cada dragao estaria vendo pelo menos 2 com olhos 
verdes, e Amadeus estava vendo nao apenas 2, mas 3 dragoes com olhos 
verdes!

Sera' que alguem vira passarinho nesssa historia ???

Grande abraco,
Rogerio Ponce


ralonso [EMAIL PROTECTED] escreveu:   Rogerio Ponce wrote: Ola' 
pessoal, Amadeus, Bertrand, Claude e Debret eram dragoes de olhos verdes 
(sim, existem dragoes de olhos verdes!) que viviam nos poroes de um velho 
castelo.
Eles sabiam que a maioria dos dragoes tinha olhos marrons, mas estes 
quatro, por viverem nas sombras, nao faziam ideia da cor dos proprios 
olhos.
Jamais conversavam entre si, mas cada um se julgava o mais burrinho 
dentre os quatro, embora tivesse a certeza de ser considerado o mais 
inteligente.
O fato e' que Amadeus era capaz de tirar todas as conclusoes possiveis 
exatamente 1 segundo apos receber a ultima informacao, enquanto Bertrand 
gastava 2 segundos, Claude levava 3 segundos e Debret precisava de 4 
segundos.
Um dia, Franz - a coruja - chamou-os ao ensolarado jardim externo, e 
anunciou o ultimo feitico do festival de magicos:
 Todo dragao que deduzir que tem olhos verdes imediatamente se 
transformara' em passarinho.

E acrescentou em seguida:
 O surpreendente e' que, neste instante, cada um de voces esta' vendo 
pelo menos 2 dragoes com olhos verdes.


  Se nenhum dragão deduzir que tem olhos verdes então ninguém todos 
continuarão
no mesmo estado de ser dragão.  Para que haja mudança de estado, isto é, 
para que um dragão
vire passarinho, é necessário que algum dos dragões deduza que tem olhos 
verdes. Essa dedução
ocorre por causa da própria frase da coruja, que implica que alguém tem 
olho verde:
  ...cada um de voces ...Veja, se não fosse esse ..cada um de vocês 
..  qualquer dragão, poderia incorrer
em erro, concluindo que tem olhos verdes.  Essa possibilidade não existe, 
logo a conclusão
de que alguém tem olho verde é certa.  Mas essa dedução não  pode 
contrariar o

princípio do dragão mais rápido.
   Logo o primeiro que acertadamente conclui que
tem olhos verdes é o Amadeus e  se torna passarinho após 1 segundo 
(assumindo que

o racicínio de todos é instantâneo)..
Ok.  Restaram 3 dragões e cada um deles conforme Fraz,
a coruja dissera, enxerga dois dragões de olhos verdes.  A coruja vê 3 
dragões, e portanto ela
também vê pelo menos dois dragões.  Isso está consistente com a profecia da 
coruja.
   Agora certamente, como os dragões estão se enxergando existem mais 
três dragões que

tem olho verde na história (porque?).
  Todo mundo vai concluir no final das contas que tem olho verde, 
porque

cada um dos três dragões restantes enxerga dois com olhos verdes.
 Dois segundos depois Bertrand conclui, da mesma forma que Amadeus,
que ele possui olhos verdes, pois se isso não acontecesse, contrariaria a 
hipótese de
Bertrand ser o mais rápido dentre os restantes e também contrariaria a 
hipótese de haverem

mais três dragões com olhos esverdeados.  Bertrand vira passarinho.
Se passaram 3 segundos até então.  Temos dois dragões e dois 
passarinhos. Um segundo
depois (4 segundos)  Claude conclui que possui olhos verdes e mais um 
segundo depois (5 segundos)

Debret conclui que tem olho verde e vira passarinho.



Falou isso, e espirrou por 10 segundos. Quando finalmente reabriu os olhos, 
o que Franz encontrou?
   Conclusão depois de 10 

Re: [obm-l] An�lise combinat�ria - n�mero de lutas

2007-07-20 Por tôpico Qwert Smith

Acho que o problema e bem mais simples que isso.

Para que um lutador seja eliminado ele perde 3 vezes.  Para que 9 lutadores 
sejam eliminados sao necessarias pelo menos 9 x 3 lutas.


Logo o minimo e 27.

O numero de lutas e sempre 27 + n.  'n' e o numero de lutas que o campeao 
perdeu.  Mas o campeao so pode perder no maximo 2 lutas ou nao seria o 
campeao.  Logo o maximo de lutas e 29.




From: [EMAIL PROTECTED]
Reply-To: obm-l@mat.puc-rio.br
To: obm-l@mat.puc-rio.br
Subject: Re: [obm-l]  Análise combinatória - número de lutas
Date: Fri, 20 Jul 2007 08:15:50 -0400





Tentativa
 
    Bem, duas considerações preliminares: 1) 1 é imbatível; 2) Alguns 
outros sempre perdem. Estamos assim em busca do mínimo.

    1 ganha de 2. 2 perde de 3. 2 perde de 4. 2 está fora.
   1 ganha de 5. 5 perde de 6. 5 perde de 7. 5 está fora.
    1 ganha de 8. 8 perde de 9. 8 perde de 10.8 está fora.
    9 lutas. Restam 7 contentores. Renumerando-os, temos:
    1 ganha de 2. 2 perde de 3. 2 perde de 4. 2 está fora.
   1 ganha de 5. 5 perde de 6. 5 perde de 7. 5 está fora.
    15 lutas acumuladas. 5 contentores:
    1 ganha de 2. 2 perde de 3. 2 perde de 4. 2 está fora.
    1 ganha de 5. 5 perde de 3. 5 perde de 4. 5 está fora.
    1 ganha de 2. 2 perde de 3. 2 está fora. Aqui, com três lutadores, 
razoável parece a quebra da regra: 2 saiu com duas derrotas.
    1 ganha de 2. 2 está fora. 1 é o campeão. Houve: 24 jogos. Esse é o 
mínimo.
    Agora, vamos a busca do máximo... (parece mais difícil). Bem, se 
distribuirmos o mais igualitariamente vitórias e derrotas, então, 
atingiremos o máximo, cremos. Logo: 1 ganha de 2, que perde de 3, que perde 
de 4, ... Hum: é um ciclo, com o ponteiro D (de derrota) apontando para os 
jogadores. O torneio acaba quando cada jogador é apontado três vezes, com 
exceção de um, que é apontado duas vezes. Logo, a resposta é: 9.3 + 1.2 = 
29.

    Fácil é inferir uma regra geral para o máximo, mas é para o mínimo?
    Bem, para o mínimo, vejamos: colocando-os em linha reta, e 
renumerando-os a cada três jogos, ao final dos quais o segundo sempre sai, 
até que fiquem três jogadores, a partir de quando, com três contendas acaba 
o torneio. Então, uma regra geral para n jogadores é 3(n-3) + 3.

 
Fraternalmente, João.
 
 
 
 
 

 


Estou com duvidas neste problema, gostaria de propo-lo aos  colegas.


Em um torneio de judo hah 10 contendores. Cada luta prossegue ateh que os 
jurados declarem um vencedor, nunca hah empate. O contendor que perder 3 
vezes (seguidas ou nao) eh eliminado. O torneio prossegue ateh que reste um 
unico contendor, que eh, entao, declarado campeao. Seja n o numero de lutas 
realizadas ateh a declaracao do campeao. Qual o menor e qual o maior valor 
que n pode assumir?



Abracos
Artur

=
Instruções para entrar na lista, sair da lista e usar a lista em
http://www.mat.puc-rio.br/~nicolau/olimp/obm-l.html
=Instruções 
para entrar na lista, sair da lista e usar a lista em

http://www.mat.puc-rio.br/~nicolau/olimp/obm-l.html



_
Don't get caught with egg on your face. Play Chicktionary!  
http://club.live.com/chicktionary.aspx?icid=chick_hotmailtextlink2


=
Instruções para entrar na lista, sair da lista e usar a lista em
http://www.mat.puc-rio.br/~nicolau/olimp/obm-l.html
=


Re: [obm-l] An�lise combinat�ria - n�mero de lutas

2007-07-20 Por tôpico Qwert Smith

??? de onde vc tirou 3(n-3)+3 pra minimo.

Para eliminarmos n-1 participantes numa competicao onde a elimicao se da com 
d derrotas sao necessarias (n-1)*d partidas.



From: [EMAIL PROTECTED]
Reply-To: obm-l@mat.puc-rio.br
To: obm-l@mat.puc-rio.br
Subject: Re: [obm-l]  Análise combinatória - número de lutas
Date: Fri, 20 Jul 2007 09:48:53 -0400

Bem, encontramos: mínimo: 24. Máximo: 29.E ainda, as regras gerais: mínimo: 
3(n-3)+3  máximo: (n-1)*3+ 2

[EMAIL PROTECTED] escreveu: -

Para: obm-l@mat.puc-rio.br
De: Qwert Smith [EMAIL PROTECTED]
Enviado por: [EMAIL PROTECTED]
Data: 20/07/2007 8:36
Assunto: Re: [obm-l] Análise combinatória - número de lutas

Acho que o problema e bem mais simples que isso.

Para que um lutador seja eliminado ele perde 3 vezes.  Para que 9 lutadores
sejam eliminados sao necessarias pelo menos 9 x 3 lutas.

Logo o minimo e 27.

O numero de lutas e sempre 27 + n.  'n' e o numero de lutas que o campeao
perdeu.  Mas o campeao so pode perder no maximo 2 lutas ou nao seria o
campeao.  Logo o maximo de lutas e 29.


From: [EMAIL PROTECTED]
Reply-To: obm-l@mat.puc-rio.br
To: obm-l@mat.puc-rio.br
Subject: Re: [obm-l]  Análise combinatória - número de lutas
Date: Fri, 20 Jul 2007 08:15:50 -0400





Tentativa
 
    Bem, duas considerações preliminares: 1) 1 é imbatível; 2) Alguns
outros sempre perdem. Estamos assim em busca do mínimo.
    1 ganha de 2. 2 perde de 3. 2 perde de 4. 2 está fora.
   1 ganha de 5. 5 perde de 6. 5 perde de 7. 5 está fora.
    1 ganha de 8. 8 perde de 9. 8 perde de 10.8 está fora.
    9 lutas. Restam 7 contentores. Renumerando-os, temos:
    1 ganha de 2. 2 perde de 3. 2 perde de 4. 2 está fora.
   1 ganha de 5. 5 perde de 6. 5 perde de 7. 5 está fora.
    15 lutas acumuladas. 5 contentores:
    1 ganha de 2. 2 perde de 3. 2 perde de 4. 2 está fora.
    1 ganha de 5. 5 perde de 3. 5 perde de 4. 5 está fora.
    1 ganha de 2. 2 perde de 3. 2 está fora. Aqui, com três 
lutadores,

razoável parece a quebra da regra: 2 saiu com duas derrotas.
    1 ganha de 2. 2 está fora. 1 é o campeão. Houve: 24 jogos. Esse é 
o

mínimo.
    Agora, vamos a busca do máximo... (parece mais difícil). Bem, se
distribuirmos o mais igualitariamente vitórias e derrotas, então,
atingiremos o máximo, cremos. Logo: 1 ganha de 2, que perde de 3, que 
perde
de 4, ... Hum: é um ciclo, com o ponteiro D (de derrota) apontando para 
os

jogadores. O torneio acaba quando cada jogador é apontado três vezes, com
exceção de um, que é apontado duas vezes. Logo, a resposta é: 9.3 + 1.2 =
29.
    Fácil é inferir uma regra geral para o máximo, mas é para o 
mínimo?

    Bem, para o mínimo, vejamos: colocando-os em linha reta, e
renumerando-os a cada três jogos, ao final dos quais o segundo sempre 
sai,
até que fiquem três jogadores, a partir de quando, com três contendas 
acaba

o torneio. Então, uma regra geral para n jogadores é 3(n-3) + 3.
 
Fraternalmente, João.
 
 
 
 
 

 


Estou com duvidas neste problema, gostaria de propo-lo aos  colegas.


Em um torneio de judo hah 10 contendores. Cada luta prossegue ateh que os
jurados declarem um vencedor, nunca hah empate. O contendor que perder 3
vezes (seguidas ou nao) eh eliminado. O torneio prossegue ateh que reste 
um
unico contendor, que eh, entao, declarado campeao. Seja n o numero de 
lutas
realizadas ateh a declaracao do campeao. Qual o menor e qual o maior 
valor

que n pode assumir?


Abracos
Artur

=
Instruções para entrar na lista, sair da lista e usar a lista em
http://www.mat.puc-rio.br/~nicolau/olimp/obm-l.html
=Instruções
para entrar na lista, sair da lista e usar a lista em
http://www.mat.puc-rio.br/~nicolau/olimp/obm-l.html


_
Don't get caught with egg on your face. Play Chicktionary! 
http://club.live.com/chicktionary.aspx?icid=chick_hotmailtextlink2

=
Instruções para entrar na lista, sair da lista e usar a lista em
http://www.mat.puc-rio.br/~nicolau/olimp/obm-l.html
=

Instruções 
para entrar na lista, sair da lista e usar a lista em

http://www.mat.puc-rio.br/~nicolau/olimp/obm-l.html



_
http://imagine-windowslive.com/hotmail/?locale=en-usocid=TXT_TAGHM_migration_HM_mini_pcmag_0507

Re: [obm-l] An�lise combinat�ria - n�mero de lutas

2007-07-20 Por tôpico Qwert Smith
Okagora entendi.  Vc escolheu dar a solucao pra uma versao adaptada do 
problema e nao ao problema proposto.  Infelizmente eu nao sou advinho e se 
vc tivesse avisado antes, eu nao teria te corrigido...foi mal ae




From: [EMAIL PROTECTED]
Reply-To: obm-l@mat.puc-rio.br
To: obm-l@mat.puc-rio.br
Subject: Re: [obm-l]  Análise combinatória - número de lutas
Date: Fri, 20 Jul 2007 11:32:17 -0400

É que o problema necessita de uma retificação. Quando se chega a 3 
participantes, duas disputas bastam para eliminar 1. E, com 2 
participantes, basta uma disputa para eliminar o perdedor e definir o 
vencedor.

[EMAIL PROTECTED] escreveu: -

Para: obm-l@mat.puc-rio.br
De: Qwert Smith [EMAIL PROTECTED]
Enviado por: [EMAIL PROTECTED]
Data: 20/07/2007 10:13
Assunto: Re: [obm-l] Análise combinatória - número de lutas

??? de onde vc tirou 3(n-3)+3 pra minimo.

Para eliminarmos n-1 participantes numa competicao onde a elimicao se da 
com

d derrotas sao necessarias (n-1)*d partidas.

From: [EMAIL PROTECTED]
Reply-To: obm-l@mat.puc-rio.br
To: obm-l@mat.puc-rio.br
Subject: Re: [obm-l]  Análise combinatória - número de lutas
Date: Fri, 20 Jul 2007 09:48:53 -0400

Bem, encontramos: mínimo: 24. Máximo: 29.E ainda, as regras gerais: 
mínimo:

3(n-3)+3  máximo: (n-1)*3+ 2
[EMAIL PROTECTED] escreveu: -

Para: obm-l@mat.puc-rio.br
De: Qwert Smith [EMAIL PROTECTED]
Enviado por: [EMAIL PROTECTED]
Data: 20/07/2007 8:36
Assunto: Re: [obm-l] Análise combinatória - número de lutas

Acho que o problema e bem mais simples que isso.

Para que um lutador seja eliminado ele perde 3 vezes.  Para que 9 
lutadores

sejam eliminados sao necessarias pelo menos 9 x 3 lutas.

Logo o minimo e 27.

O numero de lutas e sempre 27 + n.  'n' e o numero de lutas que o campeao
perdeu.  Mas o campeao so pode perder no maximo 2 lutas ou nao seria o
campeao.  Logo o maximo de lutas e 29.


 From: [EMAIL PROTECTED]
 Reply-To: obm-l@mat.puc-rio.br
 To: obm-l@mat.puc-rio.br
 Subject: Re: [obm-l]  Análise combinatória - número de lutas
 Date: Fri, 20 Jul 2007 08:15:50 -0400
 
 
 
 
 
 Tentativa
  
     Bem, duas considerações preliminares: 1) 1 é imbatível; 2) 
Alguns

 outros sempre perdem. Estamos assim em busca do mínimo.
     1 ganha de 2. 2 perde de 3. 2 perde de 4. 2 está fora.
    1 ganha de 5. 5 perde de 6. 5 perde de 7. 5 está fora.
     1 ganha de 8. 8 perde de 9. 8 perde de 10.8 está fora.
     9 lutas. Restam 7 contentores. Renumerando-os, temos:
     1 ganha de 2. 2 perde de 3. 2 perde de 4. 2 está fora.
    1 ganha de 5. 5 perde de 6. 5 perde de 7. 5 está fora.
     15 lutas acumuladas. 5 contentores:
     1 ganha de 2. 2 perde de 3. 2 perde de 4. 2 está fora.
     1 ganha de 5. 5 perde de 3. 5 perde de 4. 5 está fora.
     1 ganha de 2. 2 perde de 3. 2 está fora. Aqui, com três
lutadores,
 razoável parece a quebra da regra: 2 saiu com duas derrotas.
     1 ganha de 2. 2 está fora. 1 é o campeão. Houve: 24 jogos. Esse 
é

o
 mínimo.
     Agora, vamos a busca do máximo... (parece mais difícil). Bem, 
se

 distribuirmos o mais igualitariamente vitórias e derrotas, então,
 atingiremos o máximo, cremos. Logo: 1 ganha de 2, que perde de 3, que
perde
 de 4, ... Hum: é um ciclo, com o ponteiro D (de derrota) apontando para
os
 jogadores. O torneio acaba quando cada jogador é apontado três vezes, 
com
 exceção de um, que é apontado duas vezes. Logo, a resposta é: 9.3 + 1.2 
=

 29.
     Fácil é inferir uma regra geral para o máximo, mas é para o
mínimo?
     Bem, para o mínimo, vejamos: colocando-os em linha reta, e
 renumerando-os a cada três jogos, ao final dos quais o segundo sempre
sai,
 até que fiquem três jogadores, a partir de quando, com três contendas
acaba
 o torneio. Então, uma regra geral para n jogadores é 3(n-3) + 3.
  
 Fraternalmente, João.
  
  
  
  
  
 
  
 
 
 Estou com duvidas neste problema, gostaria de propo-lo aos  colegas.
 
 
 Em um torneio de judo hah 10 contendores. Cada luta prossegue ateh que 
os
 jurados declarem um vencedor, nunca hah empate. O contendor que perder 
3
 vezes (seguidas ou nao) eh eliminado. O torneio prossegue ateh que 
reste

um
 unico contendor, que eh, entao, declarado campeao. Seja n o numero de
lutas
 realizadas ateh a declaracao do campeao. Qual o menor e qual o maior
valor
 que n pode assumir?
 
 
 Abracos
 Artur
 
 
=

 Instruções para entrar na lista, sair da lista e usar a lista em
 http://www.mat.puc-rio.br/~nicolau/olimp/obm-l.html
 
=Instruções

 para entrar na lista, sair da lista e usar a lista em
 http://www.mat.puc-rio.br/~nicolau/olimp/obm-l.html

Re: [obm-l] Conjectura - Teoria dos N�meros

2007-07-17 Por tôpico Qwert Smith

Nao seria esse o pequeno teorema de fermat?

a e n tem que ser co-primos e como no caso a=2, qualquer n impar e co-primo.

Afinal o teorema de fermat ou de euler? Ou sao coisas diferentes?



From: Carlos Eddy Esaguy Nehab [EMAIL PROTECTED]
Reply-To: obm-l@mat.puc-rio.br
To: obm-l@mat.puc-rio.br
Subject: Re: [obm-l] Conjectura - Teoria dos Números
Date: Tue, 17 Jul 2007 07:36:56 -0300

Oi, Yuri,

Cuidado, Yuri, só vale a ida...  Se n é primo então  a^n = a (mod n)...

Por exemplo,   3^91 = 3 (mod 91) mas  91 é composto.
Veja que 3^6 = 1 (mod 91), logo, 3^90 =1 (mod 91)...

Abraços,
Nehab


At 15:44 16/7/2007, you wrote:

Isso é um teorema do euler: a^n = a (mod n) se e somente se n eh primo.

Iuri



On 7/16/07, Angelo Schranko mailto:[EMAIL PROTECTED] 
[EMAIL PROTECTED] wrote:

Saudações Srs.

Sou novo na lista.
Por favor me ajudam a provar (ou encontrar um contra-exemplo)
para a seguinte conjectura :

(2^(n - 1) - 1)/n é inteiro = n primo

Obrigado,
[]´s
Angelo


Novo http://yahoo.com.br/oqueeuganhocomisso+Yahoo! Cadê? - Experimente 
uma nova busca.




_
http://newlivehotmail.com

=
Instruções para entrar na lista, sair da lista e usar a lista em
http://www.mat.puc-rio.br/~nicolau/olimp/obm-l.html
=


Re: [obm-l] problema do livro

2007-06-20 Por tôpico Qwert Smith
Que eu saiba o teorema da amizade diz que num grupo de 2 ou mais pessoas 
pelo menos 2 tem exatamente o mesmo numero de amizades.  O que e diferente 
de dizer que todos os numeros de amizades aparecem em dobro.



From: vandermath [EMAIL PROTECTED]
Reply-To: obm-l@mat.puc-rio.br
To: obm-l@mat.puc-rio.br
Subject: Re: [obm-l] problema do livro Date: Wed, 20 Jun 2007 10:54:41 
-0300


Prezado Luis Lopes

A minha dúvida é a seguinte: Não pode acontecer de uma pessoa ser
desconhecida de todas, mas todas as
outras conheceram pelo menos uma pessoa? Como se esse pessoa fosse um
penetra da festa?

Um abraço,

Vanderlei


Em (14:17:58), obm-l@mat.puc-rio.br escreveu:


Sauda,c~oes,

Oi Vanderlei,

Não está dito mas supõe-se que se eu não conheço
você então você também não me conhece.

Talvez aí esteja a sua dúvida.

Um abraço,
Luís

From: Bruno França dos Reis
Reply-To: obm-l@mat.puc-rio.br
To: obm-l@mat.puc-rio.br
Subject: Re: [obm-l] problema do livro é divertido resolver problemas
Date: Wed, 13 Jun 2007 23:32:31 -0300

Talvez fosse legal vc colocar detalhes sobre esse problema... se não 
quem

não conhece o livro terá que ir atrás dele pra responder pra vc!

2007/6/13, vandermath :

Caros colegas da lista!

Não entendi a explicação do problema número 14 (o teorema da amizade) 
do

livro do Luís Lopes cujo título é: É divertido resolver
problemas. Ele diz que no caso de uma das pessoas ter zero amigos, 
pelo

menos mais uma outra pessoa terá zero amigos. Porque?
Não pode acontecer de apenas uma pessoa ter nenhum amigo? Se alguém
puder
ajudar, talvez o próprio autor, eu agradeço...

Um abraço,

Vanderlei


_
MSN Busca: fácil, rápido, direto ao ponto. http://search.msn.com.br

Instruções
para entrar na lista, sair da lista e usar a lista em
http://www.mat.puc-rio.br/~nicolau/olimp/obm-l.html


--


_
Hotmail to go? Get your Hotmail, news, sports and much more! 
http://mobile.msn.com


=
Instruções para entrar na lista, sair da lista e usar a lista em
http://www.mat.puc-rio.br/~nicolau/olimp/obm-l.html
=


RE: RES: [obm-l] Teoria dos numeros

2007-06-12 Por tôpico Qwert Smith

Estranho...meu email resposta com outra conta nao apareceu na lista.

A resposta e a mesma, e eu usei o metodo de somar a parte inteira de
[n/p] + [n/p^2] + [n/p^3] + ...

A parte interessante e mostrar que o expoente de 7 e mesmo a resposta ja que
se o expoente de 7 em N! for n entao o expoente de 2 vai ser sempre PELO 
MENOS 3n e o expoente de 3 vai ser sempre PELO MENOS 2n.


Eu nao sei mostrar isso com muita formalidade nao.  O raciocinio que eu usei 
foi assim:
O expoente de 7 aumenta em 1 cada vez que N aumenta em 7, logo N aumentou em 
mais que 6 e portanto o expoente de 2 aumentou pelo menos de 3 e o expoente 
de 3 aumentou de pelo menos 2.




From: Artur Costa Steiner [EMAIL PROTECTED]
Reply-To: obm-l@mat.puc-rio.br
To: obm-l@mat.puc-rio.br
Subject: RES: [obm-l] Teoria dos numeros
Date: Tue, 12 Jun 2007 13:20:44 -0300

Obrigado Paulo
Abraços
Artur

-Mensagem original-
De: [EMAIL PROTECTED] [mailto:[EMAIL PROTECTED]
nome de Paulo Santa Rita
Enviada em: terça-feira, 12 de junho de 2007 11:24
Para: obm-l@mat.puc-rio.br
Assunto: Re: [obm-l] Teoria dos numeros


Ola Carissimo Artur e demais
colegas desta lista ... OBM-L,

E facil ver que 7^4  10200  7^5. Assim, basta considerar ate 7^4. De
7 ate  10199 temos 10199 = 7 + (A-1)*7  = A = 1457 multiplos de 7.
Considerando os multiplos de 49 teriamos 10.192 = 49 + (B-1)*49 =
B=208 multiplos de 49 e com o mesmo raciocinio achamos 29 multiplos de
343(=7^3) e 4 multiplos de 2401 (= 7^4). Logo, o total de fatores 7 em
10200 ! e A + B + C + D = 1698.

Como de 1 ate 10200 existem 1 numero par ( divisivel por 2 ) a cada
dois numeros segue que ha mais que 10200 / 2 = 5100 fatores 2 e, alem
disso, 5100  3*1698 = 5094. Igualmente, como de 1 ate 10200 existem 1
numero divisivel por 3 a cada tres numeros segue que ha mais que 10200
/ 3 = 3400 fatores 3 e, alem disso, 3400  2*1698 = 3396

Segue que N = 1698 e o numero procurado.

Esta e uma solucao PARA ATROPELAR A QUESTAO, isto e, resolucao
truculenta tipo forca bruta. Nao ha inteligencia aqui. Eu precisaria
ficar receptivo para receber ideias bonitas mas estou sem tempo.

Um Abracao
Paulo Santa Rita
3,0A20,120607

Em tempo : por favor, verifique se nao cometi algum erro de calculo. O
raciocinio e correto, eu garanto

Em 11/06/07, Artur Costa Steiner[EMAIL PROTECTED] escreveu:


 Estou tentando achar uma solucoa para o seguinte, mas ainda nao 
consegui:


 Encontrar o mair valor do ineiro n=0 tal que (10200!)/(504^n) seja 
inteiro.

 Nos temos que 504 = 2^3  * 3^2 *  7, assim, o quociente sera inteiro
 enquanto 10200! contiver os primos 2, 3  e 7 com expoentes no maximo de 
3n ,
 2n e n, respectivamente. Mas nao sei se hah uma forma facil de fazer 
isso.


 Obrigado
 Artur
=
Instruções para entrar na lista, sair da lista e usar a lista em
http://www.mat.puc-rio.br/~nicolau/olimp/obm-l.html
=

=
Instruções para entrar na lista, sair da lista e usar a lista em
http://www.mat.puc-rio.br/~nicolau/olimp/obm-l.html
=


_
Need a break? Find your escape route with Live Search Maps. 
http://maps.live.com/default.aspx?ss=Restaurants~Hotels~Amusement%20Parkcp=33.832922~-117.915659style=rlvl=13tilt=-90dir=0alt=-1000scene=1118863encType=1FORM=MGAC01


=
Instruções para entrar na lista, sair da lista e usar a lista em
http://www.mat.puc-rio.br/~nicolau/olimp/obm-l.html
=


RE: [obm-l] exercitando

2007-05-25 Por tôpico Qwert Smith

Ja responderam a segunda questao erradamente 2 vezes na minha opniao.

a + b = 436
a = 17b + b - 1 (o resto e o MAIOR POSSIVEL)

resolvendo da b=23



From: elton francisco ferreira [EMAIL PROTECTED]

...


2 - A soma de dosi numeros é 436. Dividindo-se o maior
pelo menor, obtem-se o quociente 17 e o resto é o
maior possível. o menor dos números é?

...

_
PC Magazine’s 2007 editors’ choice for best Web mail—award-winning Windows 
Live Hotmail. 
http://imagine-windowslive.com/hotmail/?locale=en-usocid=TXT_TAGHM_migration_HM_mini_pcmag_0507


=
Instruções para entrar na lista, sair da lista e usar a lista em
http://www.mat.puc-rio.br/~nicolau/olimp/obm-l.html
=


RE: [obm-l] Ajuda com um problema de teoria dos numeros

2007-04-18 Por tôpico Qwert Smith

A resposta e que nao exite solucao?

Eu pensei assim:

5y^5 = 2(7-x^2) - logo y tem que ser par

1) y = 2r

pelo mesmo raciocinio 7-x^2 tem que ser multiplo de 5

x^2 = -5(s-2)

a equacao original fica

-10s + 20 + 160r^5 = 14,  fazendo r^5 = t

10s - 160t = 6, se x e y precisam ser inteiros, entao r,s e t precisam ser 
inteiros


Eu nao sei quase nada de teoria dos numeros, assim como qualquer outro 
assunto, entao se esta totalmente louco desculpa ae



From: Artur Costa Steiner [EMAIL PROTECTED]
Reply-To: obm-l@mat.puc-rio.br
To: obm-l@mat.puc-rio.br
Subject: [obm-l] Ajuda com um problema de teoria dos numeros
Date: Wed, 18 Apr 2007 10:25:24 -0300

Existe alguma forma pratica de determinar se a equacao (diofantina) 2x^2 
+5y^5=14 tem solucao para x e y inteiros (podendo ambos assumir valores 
positivos, nulos ou negativos)?


Antes de responderem, esclareco que este eh um problema real que ocorreu 
tentando otimizar um sistema elétrico. Alguem pode achar que nao eh correto 
pedir ajuda para problemas deste tipo.


Obrigado

Artur


_
The average US Credit Score is 675. The cost to see yours: $0 by Experian. 
http://www.freecreditreport.com/pm/default.aspx?sc=660600bcd=EMAILFOOTERAVERAGE


=
Instruções para entrar na lista, sair da lista e usar a lista em
http://www.mat.puc-rio.br/~nicolau/olimp/obm-l.html
=


RE: [obm-l] An�lise combinat�ria

2007-03-22 Por tôpico Qwert Smith

Os numeros tem que terminar em 0,4,6 ou 8

para numeros terminando em 0 temos:
6*5*4*1 opcoes
para numeros terminados em 4,6 ou 8 temos:
5*5*4*3 opcoes ( a unica diferenca e que 0 nao e uma opcao possivel pro 
primeiro digito)


Somando os 2 casos da b) 420 como resposta.


From: Bruna Carvalho [EMAIL PROTECTED]
Reply-To: obm-l@mat.puc-rio.br
To: obm-l@mat.puc-rio.br
Subject: [obm-l] Análise combinatória
Date: Thu, 22 Mar 2007 15:34:32 -0300

Os números pares com 4 algarismos distintos, que podemos obter com os
elementos do conjunto {0; 3; 4; 5; 6; 7; 8}, são em números de:

a) 6³
b) 420
c) 5.6²
d) 5.4³
e) 380

--
Bjos,
Bruna


_
Exercise your brain! Try Flexicon. 
http://games.msn.com/en/flexicon/default.htm?icid=flexicon_hmemailtaglinemarch07


=
Instruções para entrar na lista, sair da lista e usar a lista em
http://www.mat.puc-rio.br/~nicolau/olimp/obm-l.html
=


RE: [obm-l] Aritimetica

2007-02-16 Por tôpico Qwert Smith
Os numeros que vc procura sao da forma x*1000 + 388, onde x tem 4 
algarismos.


Como 1000 nao e multiplo de 388 prara que a soma seja e necessario que x 
seja multiplo de 388.

Logo a sua pergunta se resume a: Quantos multiplos de 388 tem 4 algarismos?

1000/388 = 2.alguma_coisa
1/388 = 25.alguma_coisa

Logo os multiplos de 388 com 4 algarismos sao: 3*388,4*388,...,25*388.
23 numeros ao todo.



From: Marcus Aurélio [EMAIL PROTECTED]
Reply-To: obm-l@mat.puc-rio.br
To: obm-l@mat.puc-rio.br
Subject: [obm-l] Aritimetica
Date: Fri, 16 Feb 2007 08:59:46 -0200

Alguem pode me ajudar nessa?

Quantos são os números de sete algarismos que são múltiplos de 388 e 
terminam

em 388?



=
Instruções para entrar na lista, sair da lista e usar a lista em
http://www.mat.puc-rio.br/~nicolau/olimp/obm-l.html
=


_
Mortgage rates as low as 4.625% - Refinance $150,000 loan for $579 a month. 
Intro*Terms  
https://www2.nextag.com/goto.jsp?product=10035url=%2fst.jsptm=ysearch=mortgage_text_links_88_h27f6disc=yvers=743s=4056p=5117


=
Instruções para entrar na lista, sair da lista e usar a lista em
http://www.mat.puc-rio.br/~nicolau/olimp/obm-l.html
=


Re: [obm-l] ajuda em probabilidade

2006-11-26 Por tôpico Qwert Smith
O seu problema Fabio e que o problema peda a probabilidade de se obter 3 
bolas vermelhas.  O que e diferente de AO MENOS 3 bolas vermelhas como vc 
resolveu.


Como a ordem das bolas nao importa vamos tirar primeiro so as vermelhas:

Chance de que a primeira bola vermelha seja retirada: 7/11
Chance que a segunda bola vermelha seja retirada:6/11
Chance que a terceira bola vermelha seja retirada: 5/11
Chance que a primeira bola branca seja retirada:4/11
Chance que a segunda bola branca seja retirada:   3/11

7*6*5*4*3/11^5 = .015..., aproximadamente 15%.



From: Fabio Silva [EMAIL PROTECTED]
Reply-To: obm-l@mat.puc-rio.br
To: obm-l@mat.puc-rio.br
Subject: Re: [obm-l] ajuda em probabilidade
Date: Sun, 26 Nov 2006 06:04:29 -0800 (PST)

Caro amigo creio q houve um engano. Voce deve ter
pensado em 3 bolas brancas nao?

Acabei de conferir o gabarito e da 15%, mas eu nao sei
como. O meu resultado deu 19%, considerando que podem
sair 3, 4 ou 5 bolas vermelhas penso eu.
E entao?

--- Roger [EMAIL PROTECTED] wrote:

 
  Uma urna contém 7 bolas vermelhas e 4 brancas. Ao
 se
  retirar simultaneamente 5 bolas ao acaso, qual a
  probabilidade de se obter 3 bolas vermelhas,
  aproximadamente?


 -

 O número de escolha possíveis para as três bolas
 vermelhas retirando 5
 bolas:

 (C4,3).7.6

 Possibilidades de se escolher 5 bolas em 11:
 C11,5

 P = (C4,3).7.6 / C11,5 = 4 . 7. 6 /  462 = 0,36






 
 


  Yahoo! Music Unlimited
  Access over 1 million songs.
  http://music.yahoo.com/unlimited
 

=
  Instruções para entrar na lista, sair da lista e
 usar a lista em
 
 http://www.mat.puc-rio.br/~nicolau/olimp/obm-l.html
 

=
 






Cheap talk?
Check out Yahoo! Messenger's low PC-to-Phone call rates.
http://voice.yahoo.com
=
Instruções para entrar na lista, sair da lista e usar a lista em
http://www.mat.puc-rio.br/~nicolau/olimp/obm-l.html
=


_
Fixing up the home? Live Search can help 
http://imagine-windowslive.com/search/kits/default.aspx?kit=improvelocale=en-USsource=hmemailtaglinenov06FORM=WLMTAG


=
Instruções para entrar na lista, sair da lista e usar a lista em
http://www.mat.puc-rio.br/~nicolau/olimp/obm-l.html
=


RE: [obm-l] Res: [obm-l] D�vida (Fun��o e Divisibilidade)

2006-10-07 Por tôpico Qwert Smith

Eu achei que eu ja tinha mostrado isso.

Mas eu vou tentar fazer mais obvio.

f(a+1) = f(a+2) + f(a)
f(a+2) = f(a+3) + f(a+1)

somando os dois lados
f(a+3) = - f(a)

Ou seja, a cada 3 termos a funcao muda de sinal

Se a quantidade de 3 termos (quantidade de mudancas de sinal) e impar a 
funcao acaba com sinal oposto, se nao acaba com o mesmo sinal


se x = 3*n + a, entao f(x) = f(a) se n e par e f(x) = -f(a) se n e impar.


Agora troca x por 2006 e a por 2.  Melhorou?



From: Jefferson Franca [EMAIL PROTECTED]
Reply-To: obm-l@mat.puc-rio.br
To: obm-l@mat.puc-rio.br
Subject: [obm-l] Res: [obm-l] Dúvida (Função e Divisibilidade)
Date: Sat, 7 Oct 2006 09:56:32 -0700 (PDT)

Será que não daria pra provar sua conjectura ? Dizer que f(x) = f(2) se x 
for par e que f(x) = - f(2) se x for ímpar é insuficiente, vc não acha?



- Mensagem original 
De: Qwert Smith [EMAIL PROTECTED]
Para: obm-l@mat.puc-rio.br
Enviadas: Quinta-feira, 5 de Outubro de 2006 18:54:26
Assunto: RE: [obm-l] Dúvida (Função e Divisibilidade)


Vou tentar a primeira:

f(3) = f(4) + f(2)
f(4) = f(5) + f(3)

somando os dois lados
f(5) = -f(2)

Mas
f(6) = f(7) + f(5)
f(7) = f(8) + f(6)

e somando temos
f(8)=-f(5)=f(2)


logo se x = 3n + 2,  f(x) = f(2) pra n par e f(x) = -f(2) pra n impar

2006 = 3*n + 2 com n par, logo f(2006) = f(2) = 1

From: André Smaira [EMAIL PROTECTED]
Reply-To: obm-l@mat.puc-rio.br
To: obm-l@mat.puc-rio.br
Subject: [obm-l] Dúvida (Função e Divisibilidade)
Date: Thu, 5 Oct 2006 14:04:43 -0300 (ART)

Apesar de acertar (foi meio na sorte), não consegui resolver estes dois
exercícios da Olimpíada Mineira de Matemática. Se vcs souberem resolver 
me

passem a resolucao:

   5-) Considere uma função que tem a seguinte propriedade: f(x+1) + 
f(x-1)

= f(x) com x inteiro. Se f(2) = 1, qual o valor de f(2006)?

   a) -1
   b) 0
   c) 1
   d) 2


   8-) Sabendo que n é um número natural e que a divisão de n por 5 deixa
resto 1; por 7 deixa resto 5 e por 9 também deixa resto 5, qual é o resto
da divisão (n + 2)*(n + 1)^2 por 315?

   a) 2
   b) 5
   c) 11
   d) 25

   Agradeço antecipadamente,
   André Smaira


-
  Novidade no Yahoo! Mail: receba alertas de novas mensagens no seu
celular. Registre seu aparelho agora!


=
Instruções para entrar na lista, sair da lista e usar a lista em
http://www.mat.puc-rio.br/~nicolau/olimp/obm-l.html
=



___
Novidade no Yahoo! Mail: receba alertas de novas mensagens no seu celular. 
Registre seu aparelho agora!

http://br.mobile.yahoo.com/mailalertas/





=
Instruções para entrar na lista, sair da lista e usar a lista em
http://www.mat.puc-rio.br/~nicolau/olimp/obm-l.html
=


RE: [obm-l] D�vida (Fun��o e Divisibilidade)

2006-10-05 Por tôpico Qwert Smith

Vou tentar a primeira:

f(3) = f(4) + f(2)
f(4) = f(5) + f(3)

somando os dois lados
f(5) = -f(2)

Mas
f(6) = f(7) + f(5)
f(7) = f(8) + f(6)

e somando temos
f(8)=-f(5)=f(2)


logo se x = 3n + 2,  f(x) = f(2) pra n par e f(x) = -f(2) pra n impar

2006 = 3*n + 2 com n par, logo f(2006) = f(2) = 1


From: André Smaira [EMAIL PROTECTED]
Reply-To: obm-l@mat.puc-rio.br
To: obm-l@mat.puc-rio.br
Subject: [obm-l] Dúvida (Função e Divisibilidade)
Date: Thu, 5 Oct 2006 14:04:43 -0300 (ART)

Apesar de acertar (foi meio na sorte), não consegui resolver estes dois 
exercícios da Olimpíada Mineira de Matemática. Se vcs souberem resolver me 
passem a resolucao:


  5-) Considere uma função que tem a seguinte propriedade: f(x+1) + f(x-1) 
= f(x) com x inteiro. Se f(2) = 1, qual o valor de f(2006)?


  a) -1
  b) 0
  c) 1
  d) 2


  8-) Sabendo que n é um número natural e que a divisão de n por 5 deixa 
resto 1; por 7 deixa resto 5 e por 9 também deixa resto 5, qual é o resto 
da divisão (n + 2)*(n + 1)^2 por 315?


  a) 2
  b) 5
  c) 11
  d) 25

  Agradeço antecipadamente,
  André Smaira


-
 Novidade no Yahoo! Mail: receba alertas de novas mensagens no seu 
celular. Registre seu aparelho agora!



=
Instruções para entrar na lista, sair da lista e usar a lista em
http://www.mat.puc-rio.br/~nicolau/olimp/obm-l.html
=


RE: [obm-l] idades

2006-09-25 Por tôpico Qwert Smith

As idades hj sao x e y
a cinco anos atras:
(x-5)=4(y-5)
daqui a cinco anos
(x+5)=2(y+5)

Um sistema de 2 equacoes e 2 variaveis com uma simples resposta nada 
olimpica de

x=25 e y=10



From: elton francisco ferreira [EMAIL PROTECTED]
Reply-To: obm-l@mat.puc-rio.br
To: obm-l@mat.puc-rio.br
Subject: [obm-l] idades
Date: Mon, 25 Sep 2006 17:11:31 -0300 (ART)

Há cinco anos a idade de severino era o quáduplo da
idade do seu filho. Dqaui a cinco anos será o dobro
quanto vale hoje a soma das idades de severino e seu filho?



___
Novidade no Yahoo! Mail: receba alertas de novas mensagens no seu celular. 
Registre seu aparelho agora!

http://br.mobile.yahoo.com/mailalertas/


=
Instruções para entrar na lista, sair da lista e usar a lista em
http://www.mat.puc-rio.br/~nicolau/olimp/obm-l.html
=



=
Instruções para entrar na lista, sair da lista e usar a lista em
http://www.mat.puc-rio.br/~nicolau/olimp/obm-l.html
=


RE: [obm-l] Polin�mios

2006-09-05 Por tôpico Qwert Smith

P(x) = -x + 3



From: Leandro A [EMAIL PROTECTED]
Reply-To: obm-l@mat.puc-rio.br
To: obm-l@mat.puc-rio.br
Subject: [obm-l] Polinômios
Date: Tue, 5 Sep 2006 10:07:34 -0300 (Hora oficial do Brasil)

Bom dia!

Preciso de Ajuda:

Um Polinômio P(X) tal que:

P(x) + x . P(2 - x) = x² +3

Calcular P(x), sabendo que P(x) = ax + b

Obrigado!

Leandro



=
Instruções para entrar na lista, sair da lista e usar a lista em
http://www.mat.puc-rio.br/~nicolau/olimp/obm-l.html
=


RE: [obm-l] Duvidas

2006-09-01 Por tôpico Qwert Smith

A porcentagem de homens e 99%.

Vc quer o numero de homens que tem que sair pra que a pocentagem caia pra 
98%.


Sem conta da pra ver que sao 50 homens que tem que sair, mas se vc prefere:

99-x = [98*(100-x)]/100
9900-100x=9800-98x
100=2x - x=50



From: matduvidas48 [EMAIL PROTECTED]
Reply-To: obm-l@mat.puc-rio.br
To: obm-l obm-l@mat.puc-rio.br
Subject: [obm-l] Duvidas
Date: Fri,  1 Sep 2006 08:29:30 -0300

Alguém poderia me ajudar nesta questão.


Em um auditório há 99 homens e uma mulher. Determine quantos homens devem 
deixar o auditório para que a porcentagem de homens presente, seja reduzida 
de apenas um ponto percentual.


Obrigada.

Aline Marques



=
Instruções para entrar na lista, sair da lista e usar a lista em
http://www.mat.puc-rio.br/~nicolau/olimp/obm-l.html
=


Re: [obm-l] Primo e divisor

2006-08-31 Por tôpico Qwert Smith
Eu acho que outros nao se interessaram pela questao por ela estar mal 
formulada.


Se a resposta e pra ser dada em funcao de p entao obviamente p^4 - 1 e a 
resposta.


Se a pergunta e o maior inteiro que garantidamente divide p^4-1 pra qualquer 
p primo  5 entao acho que a resposta e 10.


p^4-1  = 0 mod 2
p^4-1 =  0 mod 5
ja que pelo pequeno teorema de fermat, com a e p co-primos vale  a^(p-1) = 1 
mod p.


Logo 10 e garantidamente um divisor de p^4-1 pra qualquer p.  Mas certamente 
nao vai ser o maior divisor.




From: its matematico [EMAIL PROTECTED]
Reply-To: obm-l@mat.puc-rio.br
To: obm-l@mat.puc-rio.br
Subject: Re: [obm-l] Primo e  divisor
Date: Thu, 31 Aug 2006 15:06:13 + (GMT)

Acho q tenho uma solução razoável:

  se p é primo e p5 então p é ímpar, sendo assim p^4 é ímpar, logo p^4-1 
é par

  e sendo assim o maior inteiro q divide p^4-1 é: (p^4-1)/2

  Alguma objeção à resposta???

  Espero ter contribuído...
  Até +,
  Ítalo

João Luís Gomes Guimarães [EMAIL PROTECTED] escreveu:
  Se isso fosse uma questão de prova, eu responderia que o maior inteiro 
que

divide p^4 - 1 é.. p^4 - 1 ! e ninguém poderia colocar objeção,
hehehehehe... mas é claro, apesar de não ter sido explicitado, que a 
solução

procurada exclui o próprio p^4 - 1.

Apenas uma brincadeirinha pra descontrair. Vou pensar na solução aqui e, se
a encontrar, posto depois.

Abraços,

João Luís.


- Original Message -
From: Ricardo Khawge
To:
Sent: Thursday, August 31, 2006 8:51 AM
Subject: [obm-l] Primo e divisor


 Eu e colega estamos resolvendo alguns problemas e não conseguimos fazer
 um deles. Ele pediu ajuda mas ninguém se interessou pelo problema, não 
sei

 se é por ser muito fácil. Se puderem dar uma ajuda estamos postando aqui
 e agradecemos qualquer colaboração.

 Determine o maior inteiro que divide p^4 - 1, onde p é um primo maior 
que

 5.

 Tchau

 _
 Seja um dos primeiros a testar o novo Windows Live Mail Beta- grátis.
 Acesse
 
http://www.ideas.live.com/programpage.aspx?versionId=5d21c51a-b161-4314-9b0e-4911fb2b2e6d


 
=

 Instruções para entrar na lista, sair da lista e usar a lista em
 http://www.mat.puc-rio.br/~nicolau/olimp/obm-l.html
 
=



=
Instruções para entrar na lista, sair da lista e usar a lista em
http://www.mat.puc-rio.br/~nicolau/olimp/obm-l.html
=



-
 Yahoo! Acesso Grátis - Internet rápida e grátis. Instale o discador 
agora!



=
Instruções para entrar na lista, sair da lista e usar a lista em
http://www.mat.puc-rio.br/~nicolau/olimp/obm-l.html
=


Re: [obm-l] Primo e divisor

2006-08-31 Por tôpico Qwert Smith
Eu vi depois de apertar send que devia ter testado divisibilidade por 3 e 
mesmo sem fatorar da pra ver que p^4 - 1 = 0 mod 3, ja que p = 1 mod 3 ou p 
= -1 mod 3


Entao logo de cara eu ja devia ter mudado a resposta para pelo menos 2*3*5 = 
30.


Mas pegando carona na sua resposta (embora o merito seja seu :D ):

p^4 - 1 = (p^2 +1)(p -1 )(p+1)  que é (obviamente) divisível por 8

Ta bom 2x*2y*2z e divisivel por 8.

Mas na verdade p-1 ou p +1 são divisíveis por 4.

Entao temos 2x*4s*2z ou 2x*2y*4t o que e divisivel por 16.

Logo o maior numero garantido e 2^4*3*5 = 240, confere?


From: Carlos Eddy Esaguy Nehab [EMAIL PROTECTED]
Reply-To: obm-l@mat.puc-rio.br
To: obm-l@mat.puc-rio.br
Subject: Re: [obm-l] Primo e divisor
Date: Thu, 31 Aug 2006 17:22:05 -0300

Oi,

Você deve ter razão quanto à formulação mas trivialmente sua solução pode 
ser melhorada para 120 (embora o mérito seja seu), pois


p^4 - 1 = (p^2 +1)(p -1 )(p+1)  que é (obviamente) divisível por 8 e além 
disso, p-1 ou p+1 é divisível por 3. Mas na verdade p-1 ou p +1 são 
divisíveis por 4.  Logo... vale a melhoria 120, mas também não sei como 
melhorá-la mais um pouquinho nem poucão...


Abraços,
Nehab

At 12:47 31/8/2006, you wrote:
Eu acho que outros nao se interessaram pela questao por ela estar mal 
formulada.


Se a resposta e pra ser dada em funcao de p entao obviamente p^4 - 1 e a 
resposta.


Se a pergunta e o maior inteiro que garantidamente divide p^4-1 pra 
qualquer p primo  5 entao acho que a resposta e 10.


p^4-1  = 0 mod 2
p^4-1 =  0 mod 5
ja que pelo pequeno teorema de fermat, com a e p co-primos vale  a^(p-1) = 
1 mod p.


Logo 10 e garantidamente um divisor de p^4-1 pra qualquer p.  Mas 
certamente nao vai ser o maior divisor.




From: its matematico [EMAIL PROTECTED]
Reply-To: obm-l@mat.puc-rio.br
To: obm-l@mat.puc-rio.br
Subject: Re: [obm-l] Primo e  divisor
Date: Thu, 31 Aug 2006 15:06:13 + (GMT)

Acho q tenho uma solução razoável:

  se p é primo e p5 então p é ímpar, sendo assim p^4 é ímpar, logo 
p^4-1 é par

  e sendo assim o maior inteiro q divide p^4-1 é: (p^4-1)/2

  Alguma objeção à resposta???

  Espero ter contribuído...
  Até +,
  Ítalo

João Luís Gomes Guimarães [EMAIL PROTECTED] escreveu:
  Se isso fosse uma questão de prova, eu responderia que o maior inteiro 
que

divide p^4 - 1 é.. p^4 - 1 ! e ninguém poderia colocar objeção,
hehehehehe... mas é claro, apesar de não ter sido explicitado, que a 
solução

procurada exclui o próprio p^4 - 1.

Apenas uma brincadeirinha pra descontrair. Vou pensar na solução aqui e, 
se

a encontrar, posto depois.

Abraços,

João Luís.


- Original Message -
From: Ricardo Khawge
To:
Sent: Thursday, August 31, 2006 8:51 AM
Subject: [obm-l] Primo e divisor


 Eu e colega estamos resolvendo alguns problemas e não conseguimos 
fazer
 um deles. Ele pediu ajuda mas ninguém se interessou pelo problema, não 
sei
 se é por ser muito fácil. Se puderem dar uma ajuda estamos postando 
aqui

 e agradecemos qualquer colaboração.

 Determine o maior inteiro que divide p^4 - 1, onde p é um primo maior 
que

 5.

 Tchau

 _
 Seja um dos primeiros a testar o novo Windows Live Mail Beta- grátis.
 Acesse
 
http://www.ideas.live.com/programpage.aspx?versionId=5d21c51a-b161-4314-9b0e-4911fb2b2e6d


 
=

 Instruções para entrar na lista, sair da lista e usar a lista em
 http://www.mat.puc-rio.br/~nicolau/olimp/obm-l.html
 
=



=
Instruções para entrar na lista, sair da lista e usar a lista em
http://www.mat.puc-rio.br/~nicolau/olimp/obm-l.html
=



-
 Yahoo! Acesso Grátis - Internet rápida e grátis. Instale o discador 
agora!



=
Instruções para entrar na lista, sair da lista e usar a lista em
http://www.mat.puc-rio.br/~nicolau/olimp/obm-l.html
=


=
Instruções para entrar na lista, sair da lista e usar a lista em
http://www.mat.puc-rio.br/~nicolau/olimp/obm-l.html
=



=
Instruções para entrar na lista, sair da lista e usar a lista em
http://www.mat.puc-rio.br/~nicolau/olimp/obm-l.html
=


[obm-l] Teoria dos numeros?

2006-08-01 Por tôpico Qwert Smith

Liste todos os pares (m,n) para os quais 2^m + 3^n e um quadrado perfeito.


=
Instruções para entrar na lista, sair da lista e usar a lista em
http://www.mat.puc-rio.br/~nicolau/olimp/obm-l.html
=


Re:[obm-l] Primos gemeos

2006-06-01 Por tôpico Qwert Smith

a^p - a = 1 tb resulta em 2(a^p - a) + 3  primo.

Se os primos p e q sao primos gemeos e pq entao
p= 6k - 1 e q 6k + 1

Logo o problema se resume a provar que 2(a^p - a + 1) nunca sera um multiplo 
de 6.

Mas o Claudio ja mostrou que a^p - a = 3t.  2(3t + 1) = 2 (mod 6).

Vale assim?


From: claudio\.buffara [EMAIL PROTECTED]
Reply-To: obm-l@mat.puc-rio.br
To: obm-l obm-l@mat.puc-rio.br
Subject: Re:[obm-l] Primos gemeos
Date: Thu,  1 Jun 2006 09:49:11 -0300

-- Cabeçalho original ---

De: [EMAIL PROTECTED]
Para: obm-l@mat.puc-rio.br
Cópia:
Data: Wed, 31 May 2006 19:36:57 -0700 (PDT)
Assunto: [obm-l] Primos gemeos

 Este problema que me foi proposto me pareceu
 interessante:

 Mostre que, se a e p forem inteiros positivos com p
 impar, entao o numero 2(a^p - a + 1) nunca estah
 compreendido entre 2 primos gemeos.

 Artur



Como p eh impar a^p - a eh sempre divisivel por 3, pois:
a == 0, 1, 2 (mod 3) == a^p == 0, 1, 2 (mod 3).
Logo, 2(a^p - a) + 3 eh multiplo de 3 e soh serah primo se a^p = a.
Mas nesse caso, 2(a^p - a) + 1 = 1, que nao eh primo.

[]s,
Claudio.



=
Instruções para entrar na lista, sair da lista e usar a lista em
http://www.mat.puc-rio.br/~nicolau/olimp/obm-l.html
=



=
Instruções para entrar na lista, sair da lista e usar a lista em
http://www.mat.puc-rio.br/~nicolau/olimp/obm-l.html
=


RE: [obm-l] Quest�o de concurso

2006-04-10 Por tôpico Qwert Smith

Se d, t e q sao o numero de duplos, tripos e quadruplos entao
1) d = t = q

Se r sao os filhos que nao sao gemeos entao
2) 39 - r = 2d ou 2t ou 2q
Mas se d =2x, t =3y, q = 4z entao
39 - r = 24w (4x, 6y, 8z - mmc 4,6,8 = 24 )
como r  0 = w = 1 e r - 15
2d=2t=2q=24 = d=t=q=12
6 duplas de gemeos, 4 trincas de trigemeos, 3 grupos de quadrigemeos e um 
total de 51 filhos.



From: Alexandre Bastos [EMAIL PROTECTED]
Reply-To: obm-l@mat.puc-rio.br
To: OBM obm-l@mat.puc-rio.br
Subject: [obm-l] Questão de concurso
Date: Mon, 10 Apr 2006 13:47:43 + (GMT)

Bom dia, amigos. Alguém pode me ajudar com esse probleminha?

  O emir Abdel Azir ficou famoso por vários motivos. Ele teve mais de 39 
filhos, incluindo muitos gêmeos. De fato, o historiador Ahmed Aab afirma 
num dos seus escritos que todos os filhos do emir eram gêmeos duplos, 
exceto 39; todos eram gêmeos triplos, exceto 39; todos eram gêmeos 
quádruplos, exceto 39. O numero de filhos do emir é:



-
 Abra sua conta no Yahoo! Mail - 1GB de espaço, alertas de e-mail no 
celular e anti-spam realmente eficaz.



=
Instruções para entrar na lista, sair da lista e usar a lista em
http://www.mat.puc-rio.br/~nicolau/olimp/obm-l.html
=


RE: [obm-l] PA e primos

2006-03-29 Por tôpico Qwert Smith

O limite superior e provavelmente o da sua prova.

a_(a1+1) = a1 + a1 * r = a1 * ( r + 1 ) e nao primo

Mas esse e o pior caso possivel.  Seja p um primo tal que r = 1 mod p
Entao o primeiro termo composto sera a1 + n * r onde n = p - [a1 mod p]

foi o que me veio a cabeca ate agora



From: Iuri [EMAIL PROTECTED]
Reply-To: obm-l@mat.puc-rio.br
To: obm-l@mat.puc-rio.br
Subject: [obm-l] PA e primos
Date: Wed, 29 Mar 2006 10:24:50 -0300

Vi uma questão esses dias, e fiquei curioso.

Em uma PA infinita, com termo inicial a1 natural e razão r natural e
diferente de zero. Prove que esta PA não pode ser composta apenas por
numeros primos.

Consegui provar, mas fiquei um tempão pensando: em qual termo exatamente
acontece a primeira ocorrencia de um numero nao primo, em função de r e a1?
Isso é uma questao razoavel ou vai ter q desenvolver mta coisa pra chegar 
no

resultado?

Iuri



=
Instruções para entrar na lista, sair da lista e usar a lista em
http://www.mat.puc-rio.br/~nicolau/olimp/obm-l.html
=


Re: [obm-l] T. Numeros

2006-03-28 Por tôpico Qwert Smith


Eu acho que a pergunta pode ser uma derivada dessa aqui.  Prove que 
qualquer numero pode ser escrito com no maximo 5 numeros piramidais


http://www2.toki.or.id/book/AlgDesignManual/BOOK/BOOK/NODE38.HTM


From: claudio\.buffara [EMAIL PROTECTED]
Reply-To: obm-l@mat.puc-rio.br
To: obm-l obm-l@mat.puc-rio.br
Subject: Re: [obm-l] T. Numeros
Date: Tue, 28 Mar 2006 17:49:07 -0300

Se os cubos tiverem que ser não-negativos, então isso é falso.
Tente expressar 23 como soma de cubos.
O mínimo número de cubos não-negativos necessário pra expressar qualquer 
inteiro positivo (como uma soma de cubos) é 9 e, se você tiver uma prova 
por indução desse fato, eu gostaria muito de vê-la.


Por outro lado, se os cubos puderem ser negativos, então:
23 = 27 + (-1) + (-1) + (-1) + (-1).
No entanto, eu não sei se 5 cubos são sempre suficientes.

Procure Waring's problem no Google.

[]s,
Claudio.

De:[EMAIL PROTECTED]

Para:obm-l@mat.puc-rio.br

Cópia:

Data:Tue, 28 Mar 2006 15:15:10 -0300

Assunto:Re: [obm-l] T. Numeros

 Todo inteiro, ou todo inteiro maior que 5?
 Para todos os inteiros menores que 5 basta tomar os
 primeiros cubos iguais a zero:

 1 = 0^3 + 0^3 + 0^3 + 0^3 + 1^3 , etc ...

 Para inteiros maiores que 5, deve haver algum truque que
 permita concluir que se n se escreve como soma de cubos
 então n+1 também se escreve como a soma de cubos.
 Daía a prova sai por indução.

- Original Message -
From: Klaus Ferraz
To: obm-l@mat.puc-rio.br
Sent: Friday, March 17, 2006 11:57 PM
Subject: [obm-l] T. Numeros


Mostre que todo inteiro pode ser escrito como soma de 5 cubos.



=
Instruções para entrar na lista, sair da lista e usar a lista em
http://www.mat.puc-rio.br/~nicolau/olimp/obm-l.html
=


RE: [obm-l] teoria combinatoria dos numeros(?) [Era: probleminhas]

2006-03-09 Por tôpico Qwert Smith


Uma coisa interessante de notar (ou nao) e que xy - x - y e quase xy - x - y 
+ 1


logo x.y - (x+y) = (x-1)(y-1) - 1

Re-escrevendo isso... Se temos moedas p e q podemos fazer todo tipo de troco 
de (p-1)(q-1) pra cima desde que o numero de moedas a disposicao seja 
infinito.  Pra quem ja viu crioptografia rsa percebe n=pq e m=(p-1)(q-1).  
Nao sei se e pura coincidencia ou nao.  Talvez aquele colega que jura ter 
quebrado o RSA pode elucidar alguma coisa.  Eu de minha parte prefiro 
abandonar o problema por aqui pra nao ser abduzido por seres extraterrestres 
ou agentes da cia.





From: Luís Lopes [EMAIL PROTECTED]
Reply-To: obm-l@mat.puc-rio.br
To: obm-l@mat.puc-rio.br
Subject: [obm-l] teoria combinatoria dos numeros(?) [Era: probleminhas]
Date: Wed, 08 Mar 2006 20:38:08 +

Sauda,c~oes,

Discuto esse problema (ou melhor, a fórmula)


número maximo = X . Y - ( X + Y )


na solução do problema 15 do livro É divertido resolver problemas.
Ver o link

http://www.escolademestres.com/qedtexte/sol1.pdf


alguem sabe provar isso???

Ou refutar??

Também não sei.

[]'s
Luís



From: Felipe Avelino [EMAIL PROTECTED]
Reply-To: obm-l@mat.puc-rio.br
To: obm-l@mat.puc-rio.br
Subject: Re: [obm-l] probleminhas
Date: Wed, 8 Mar 2006 16:05:59 -0300

isso se torna muito cansativo no caso de um numero muito grande...

existe uma forma que se eu me recordo eh...no caso de dois numeros X e Y
primos entre si.. que eh

número maximo = X . Y - ( X + Y )

alguem sabe provar isso???
deve envolver teoria combinatoria dos numeros .. não sei ..

Em 08/03/06, João Gilberto Ponciano Pereira [EMAIL PROTECTED]
escreveu:

 Cheguei em 23...

 A lógica que usei é a seguinte Temos que conseguir o menor número 
das

 unidades. Após isso, basta somar 2 vezes a cota de 2 bombons de 5.

 Temos então que achar a combinação de bombons tal que o total seja o
 mínimo, para cada uma das unidades. Temos então:

 0 == 0  = 0x5 + 0x7
 1 == 21 = 0x5 + 3x7
 2 == 12 = 1x5 + 1x7
 3 == 33 = 1x5 + 4x7
 4 == 14 = 0x5 + 2x7
 5 == 05 = 1x5 + 0x7
 6 == 26 = 1x5 + 3x7
 7 == 07 = 0x5 + 1x7
 8 == 28 = 0x5 + 4x7
 9 == 19 = 1x5 + 2x7

 logo.. como o maior desta lista é o 33, se subtrairmos 10, temos que o
 maior número de bombons que não se pode vender com a combinação de 5 e 
7

 bombons é 23.


 -Original Message-
 From: [EMAIL PROTECTED] [mailto:[EMAIL PROTECTED]
 Behalf Of Henrique Ren
 Sent: Wednesday, March 08, 2006 1:28 PM
 To: obm-l@mat.puc-rio.br
 Subject: [obm-l] probleminhas


 Encontrei esse probleminha e gostaria que alguém me ajudasse a 
resolvê-lo:


 uma doceria venda caixas com 05 e 07 bombons dentro. qual o número 
máximo

 de
 bombons que a doceria não consegue vender?
 por exemplo: consegue-se vender 17 bombons porém não 11 bombons?

 []s



=
Instruções para entrar na lista, sair da lista e usar a lista em
http://www.mat.puc-rio.br/~nicolau/olimp/obm-l.html
=



=
Instruções para entrar na lista, sair da lista e usar a lista em
http://www.mat.puc-rio.br/~nicolau/olimp/obm-l.html
=


RE: [obm-l] probleminhas

2006-03-08 Por tôpico Qwert Smith

Eu fiz assim

Se n pode ser reprensetado entao n+1 tb pode ser representado de 2 maneiras:

n - 4*5 + 3*7 ou n - 2*7 + 3*5

Ou seja, se n = (4+)*5 + (2+)*7 entao sempre e possivel escrever n+1.

Ja pra n= 3*5 +1*7 = 22,  nao podemos modificar pra representarmos 23



From: João Gilberto Ponciano Pereira [EMAIL PROTECTED]
Reply-To: obm-l@mat.puc-rio.br
To: obm-l@mat.puc-rio.br
Subject: RE: [obm-l] probleminhas
Date: Wed, 8 Mar 2006 15:29:02 -0300

Cheguei em 23...

A lógica que usei é a seguinte Temos que conseguir o menor número das 
unidades. Após isso, basta somar 2 vezes a cota de 2 bombons de 5.


Temos então que achar a combinação de bombons tal que o total seja o 
mínimo, para cada uma das unidades. Temos então:


0 == 0  = 0x5 + 0x7
1 == 21 = 0x5 + 3x7
2 == 12 = 1x5 + 1x7
3 == 33 = 1x5 + 4x7
4 == 14 = 0x5 + 2x7
5 == 05 = 1x5 + 0x7
6 == 26 = 1x5 + 3x7
7 == 07 = 0x5 + 1x7
8 == 28 = 0x5 + 4x7
9 == 19 = 1x5 + 2x7

logo.. como o maior desta lista é o 33, se subtrairmos 10, temos que o 
maior número de bombons que não se pode vender com a combinação de 5 e 7 
bombons é 23.



-Original Message-
From: [EMAIL PROTECTED] [mailto:[EMAIL PROTECTED]
Behalf Of Henrique Ren
Sent: Wednesday, March 08, 2006 1:28 PM
To: obm-l@mat.puc-rio.br
Subject: [obm-l] probleminhas


Encontrei esse probleminha e gostaria que alguém me ajudasse a resolvê-lo:

uma doceria venda caixas com 05 e 07 bombons dentro. qual o número máximo 
de

bombons que a doceria não consegue vender?
por exemplo: consegue-se vender 17 bombons porém não 11 bombons?

[]s


=
Instruções para entrar na lista, sair da lista e usar a lista em
http://www.mat.puc-rio.br/~nicolau/olimp/obm-l.html
=

=
Instruções para entrar na lista, sair da lista e usar a lista em
http://www.mat.puc-rio.br/~nicolau/olimp/obm-l.html
=



=
Instruções para entrar na lista, sair da lista e usar a lista em
http://www.mat.puc-rio.br/~nicolau/olimp/obm-l.html
=


RE: [obm-l] Re: [obm-l] Re: [obm-l] Re: [obm-l] Fw: congru�ncia

2006-03-05 Por tôpico Qwert Smith
Talvez uma maneira menos tecnica mas que pelo menos pra mim e mais facil de 
ver e a seguinte:


10^2 = 8 mod 23

Queremos todo k tal que 10^k = 8 mod 23

n^(a+b) = n^a * n^b

se k = 2 + t entao 10^k = 10^2 * 10^t e 10^k mod 23 = 8 * 10^t mod 23

o problema entao se reduz a encontrar todos valores de t para os quais 10^t 
= 1 mod 23


Como eu tb nao entendo quase nada de congruencias eu sempre procuro chegar 
em a ~= 1 mod b que em geral da pra se resolver com conhecimentos minimos




From: Marcelo Salhab Brogliato [EMAIL PROTECTED]
Reply-To: obm-l@mat.puc-rio.br
To: obm-l@mat.puc-rio.br
Subject: [obm-l] Re: [obm-l] Re: [obm-l] Re: [obm-l] Fw: congruência
Date: Sun, 5 Mar 2006 00:20:37 -0300

Opa,
bem, nao consegui entender algumas coisas... se puder, por favor, da uma 
explicada um pouco melhor..


isso eu entendi:
10^11 = 10*(10^2)^5 = 10*8^5 = 10*16

mas nao entendi pq o fato de nao ser congruo a 1 (mod 23) faz com que 22 
seja o menor numero com essa propriedade!


Tbem nao entendi pq: 10^a = 10^b (mod 23) = a = b (mod 22)

abraços,
Salhab
  - Original Message -
  From: Marcio Cohen
  To: obm-l@mat.puc-rio.br
  Sent: Saturday, March 04, 2006 11:22 PM
  Subject: [obm-l] Re: [obm-l] Re: [obm-l] Fw: congruência


Como 23 eh primo, 10^22 = 1 (mod 23), e como 10^2 = 8 e 10^11 = 
10*(10^2)^5 = 10*8^5 = 10*16 != 1 (mod 23), 22 eh o menor numero com essa 
propriedade.

Logo, 10^a = 10^b (mod 23) se e somente se a = b (mod 22).
Como 10^2 = 8 (mod 23), a resposta é que os valores de k para os quais 
temos 10^k = 8 (mod 23) são exatamente os inteiros positivos que deixam 
resto 2 na divisão por 22 (2, 24, 46, ...)


  Abraços,
  Marcio


- Original Message -
From: Marcelo Salhab Brogliato
To: obm-l@mat.puc-rio.br
Sent: Saturday, March 04, 2006 7:06 PM
Subject: [obm-l] Re: [obm-l] Fw: congruência


Olá,

vc quer saber para quais valores de k temos:
10^k = 8 (mod 23), certo?

bom, temos que:
100 = 8 (mod 23)
10^(2n) = 8^n (mod 23)
isso é, para k par temos que a unica solucao é k=2 (n=1).

ainda nao consegui extender essa solucao para k impar.. estou 
tentando!


PS: sei mto pouco sobre congruencia, talvez minha solucao esteja 
errada


abraços,
Salhab


  - Original Message -
  From: Leo
  To: obm-l@mat.puc-rio.br
  Sent: Saturday, March 04, 2006 12:16 AM
  Subject: [obm-l] Fw: congruência



  - Original Message -
  From: Leo
  To: obm-l@mat.puc-rio.br
  Sent: Friday, March 03, 2006 8:11 PM
  Subject: congruência


  Como resolver a seguinte congruência
  10^k cong 8 (mod 23) ... pra k=2 eh verdadeira mas como achar o caso 
geral???



=
Instruções para entrar na lista, sair da lista e usar a lista em
http://www.mat.puc-rio.br/~nicolau/olimp/obm-l.html
=


RE: [obm-l] O N�MERO PENSADO!

2006-02-18 Por tôpico Qwert Smith


Carlinhos pensa num número ímpar positivo menor do que 100. Pedrinho se 
dispõe a descobrir que número é esse fazendo a seguinte pergunta, quantas 
vezes forem necessárias: O número que você pensou é maior, menor ou igual 
a x? Note que x é um número que Pedrinho escolhe. Quantas perguntas desse 
tipo Pedrinho poderá ter que fazer até descobrir o número pensado por 
Carlinhos?


6 vezes.   Temos 50 numeros possiveis e com cada pergunta podemos eliminar 
metade dos numeros

50 -25-13-7-4-2-1
Cada - e uma pergunta.



Peça a um amigo que pense um número natural qualquer, com quantos 
algarismos ele quiser, e que faça o seguinte: escreva o número pensado; 
mude como queira a ordem de seus algarismos, obtendo um novo número; 
diminua o menor do maior; retire um dos algarismos do resto (diferente de 
zero), e diga os demais algarismos na ordem que quiser. Em resposta, você 
dirá qual o algarismo retirado . Como explicar a mágica?


Seja R o resto da divisao do numero escolhido N por 9.
N' ( rearrango dos algarismos de N) tb deixara resto R quando dividido por 
9.

| N - N' | = 0 (mod 9)

O algarismo retirado sera 9 - (soma dos algarismos ditos (mod 9))



Um jogo de adivinhas, entre duas pessoas A e B, consiste no seguinte: A 
pensa num número de 1 a 7; a cada lance, B arrisca um número e A responde 
certo, alto ou baixo. Descreva uma estratégia para que B adivinhe o número, 
arriscando no máximo dois palpites.




primeiro palpite 4, segundo 2 ou 6



Abraços!

_
Ganhe tempo encontrando o arquivo ou e-mail que você precisa com Windows 
Desktop Search. Instale agora em  http://desktop.msn.com.br


=
Instruções para entrar na lista, sair da lista e usar a lista em
http://www.mat.puc-rio.br/~nicolau/olimp/obm-l.html
=



=
Instruções para entrar na lista, sair da lista e usar a lista em
http://www.mat.puc-rio.br/~nicolau/olimp/obm-l.html
=


[obm-l] probabilidade

2006-02-06 Por tôpico Qwert Smith
Um professor pede aos alunos que escrevam em uma folha de papel numeros de 1 
a 100.  Cabe aos alunos escolherem quantos e quais numeros cada um escreve.  
Se a classe tem 40 alunos, qual a probabilidade que 2 alunos tenham 
escolhido o mesmo sujconjunto de numeros?


E no caso generico de N peassoas escolhendo 1 subconjunto de um conjunto de 
M elementos?  Qual a probabilidade de 2 pessoas escolherem o mesmo 
subconjunto?



=
Instruções para entrar na lista, sair da lista e usar a lista em
http://www.mat.puc-rio.br/~nicolau/olimp/obm-l.html
=


Re:[obm-l] PROBABILIDADE

2006-01-13 Por tôpico Qwert Smith

a (c) e AO MENOS um rei.  Ou seja: 1 - (b).  De resto parece correto.


From: eritotutor [EMAIL PROTECTED]

a) O total de casos possíveis eh C(52,5). Como os 4 reis são extraídos, 
temos 48 casos favoráveis. Portanto P(Xa) = 48/C(52,5).


b) Devemos escolher 5 cartas dentre 48 possíveis. Portanto P(Xb) = 
C(48,5)/C(52,5).


c) Tome as seguintes decisões:
1- Escolha 1 rei
2 - Escolha 4 dentre as cartas restantes
Portanto P(Xc) = [C(4,1).C(51,4)]/C(52,5)

d) Tome as seguintes decisões:
1- Escolha 4 dentre as 80 camisas da marca A
2 - Escolha 1 dentre as camisas restantes
Portanto P(Xd) = [C(80,4).C(96,1)]/C(100,5)

[]s

De um baralho de 52 cartas, 5 são extraídas ao acaso, sem reposicao. Qual a 
probabilidade de :

 a)sairem os 4 reis
 b)nao sair nenhum rei
 c)sair ao menos um rei

 Em um loja existem 100 camisas, sendo 80 da marca A. Se 5 camisas forem 
escolhidas ao acaso, sem reposicao, qual a probabilidade de 4 serem da 
marca A?



Yahoo! doce lar. Faça do Yahoo! sua homepage.



=
Instruções para entrar na lista, sair da lista e usar a lista em
http://www.mat.puc-rio.br/~nicolau/olimp/obm-l.html
=


Re: [obm-l] probabilidade (ufrj)

2005-11-28 Por tôpico Qwert Smith
Usando o mesmo raciocinio que vc apresenta vc tb garante a vitoria com 6 
jogadas e raramente teria que lancar 10 moedas.  Ou ja teria ganho ou ja 
teria perdido antes.  Isso faria, de novo usando a sua ideia, o numero de 
resultados favoraveis tb menor.  Acho que a questao aqui e que a ordem nao 
importa e na verdade nao importa se vc para ou nao depois que ja perdeu.  A 
probabilidade de uma sequencia de 10 jogadas comecar com 5 caras e a mesma, 
quer vc pare ou continue depois.   Suponha que todas as moedas sao sempre 
lancadas ao mesmo tempo.  Faz diferenca?  So pq vc parou de contar quando 
viu 5 caras nao mudou o numero de jogadas.



From: [EMAIL PROTECTED]
Esse é o raciocínio do gabarito, mas se eu estivesse jogando esse jogo e
tirasse 5 caras nas 5 primeiras jogadas, para que me importaria jogar os
outros 5 lançamentos se eu já perdi? Da mesma forma que, numa melhor de 5,
se eu estivesse perdendo de 3 a 0, para que jogaria as partidas restantes?
E considerando que o jogo efetivamente seja interrompido se eu houver 
perdido

(e considero essa interpretação válida pq o 'até' do enunciado é ambíguo),
então o número de desfechos possíveis diminui, e a probabilidade aumenta.



=
Instruções para entrar na lista, sair da lista e usar a lista em
http://www.mat.puc-rio.br/~nicolau/olimp/obm-l.html
=


Re: [obm-l] RACIOCINIO SOLIDARIO!

2005-11-18 Por tôpico Qwert Smith
Pra mim os problemas sao 'quase' identicos.  Acho que o primeiro nao tem 
problema nenhum de aceitar a resposta.  Suponhamos que 1 galinha teve seu 
penacho arrancado... ela nao ve nenhuma galinha sem penacho, sabe que so 
pode ser ela e se mata.  Raciocinando com 2 agora.  Cada uma ve uma galinha 
sem penacho logo na primeira noite pensa que ainda esta com seu proprio 
penacho.  No dia seguinte pensa: Se a galinha sem penacho nao se matou e pq 
ela viu outra sem penacho.  Nao tem mais nenhuma que eu veja logo sou eu. E 
na noite 2 as 2 se matam.  Dai pra 7 e um pulo..ou 5 pulos se preferir


O segundo e um pouco mais complicado so pq as mulheres vivem traidas e 
felizes ate o dia que a Rainha fala alguma coisa.   Antes da Rainha falar as 
amazonas ja sabiam da existencia de traicao.  Porque a Rainha falar mudou 
algo?  Usando o primeiro problema, fica facil de entender.  Suponha que os 
gansos cortam penachos mas levam as penas e nao deixam nenhuma evidencia.  
As galinha veveram felizes e contentes ate alguem avisar elas que pelo menos 
uma galinha esta copm o penacho faltando.  Por que?  Vou considerar de novo 
o caso de 2 galinhas.  Cada uma ve uma galinha sem penacho.  Como no dia 
seguinte ela esta viva ela pensa: Hmmm, sera que ela tb ve alguma galinha 
(eu) sem penacho? Nah, com certeza ela nao ve nenhuma e portando ela nao 
sabe ainda que existe pelo menos uma galinha sem penacho.   Todas as 
galinhas sabem que tem pelo menos uma galinha sem penacho.  Alguem dizer 
isso pra elas nao vai adicionar essa informacao.  A unica informacao que 
isso adciona e que elas ficam sabendo que todas as outras galinhas agora tb 
sabem.  Ai o problema se resume ao primeiro ja resolvido.
Pelo menos essa e a unica confusao que acho que outros podem ter.  Nao sei 
se expliquei bem ou so confundi mais :).





From: Nicolau C. Saldanha [EMAIL PROTECTED]
Date: Thu, 17 Nov 2005 09:15:10 -0200

On Thu, Nov 17, 2005 at 11:17:12AM +, Jorge Luis Rodrigues e Silva Luis 
wrote:

 ...continuando nosso raciocínio, vamos aos problemas que ainda continuam
 em aberto...

 Num hipotético planeta habitado pelas galinhas de penacho, cada galinha,
 embora inteligente e observadora, enxergava o penacho das outras, menos 
o

 seu. Se, um dia uma das galinhas tivesse certeza de que seu penacho fora
 cortado, suicidar-se-ia naquela mesma noite. Elas não se comunicavam 
entre

 si. Uma noite, os gansos do planeta vizinho, sorrateiramente cortaram o
 penacho de algumas das galinhas enquanto elas dormiam. Na manhã 
seguinte,

 todas viram no chão as penas que indicavam o ataque dos gansos e na
 sétima noite algumas se suicidaram. Quantas foram e por que somente na
 sétima noite?

 Na ilha das amazonas todas as amazonas são casadas, menos a rainha. Se 
uma

 amazona descobre que seu marido a traiu ela o mata a meia-noite. Se uma
 amazona tem um caso com o marido de outra ela conta isso para todas as
 amazonas da ilha menos para a que foi traída. De fato, há muita
 traição na ilha: há 1000 amazonas casadas e 395 delas são traídas. Um
 dia a rainha se cansa disso tudo, chama todas as amazonas e diz: Há
 traição nesta ilha. O que acontece?

Os dois problemas são basicamente o mesmo, não?
Também já vi uma versão em que eram gaivotas doentes,
outra em que eram pessoas com a cara suja e ouvi falar
(mas nunca achei) que a primeira versão era sobre maridos
canibais que cozinhavam e comiam as esposas infiéis.

O segundo já foi discutido numa Eureka e, se não muito me engano,
nesta lista. As diferenças que me ocorrem de memória entre este e-mail
e o problema como saiu na Eureka é o número de maridos infiéis
(acho que eram 365) e as aspas ao redor da palavra menos,
que, a meu ver, não fazem sentido.

Já tive uma longa discussão por e-mail com um interlocutor
(que prefiro não nomear) que estava convencido de que a solução clássica
estava errada; posso mandar a explicação que escrevi para ele para a lista.

[]s, N.



=
Instruções para entrar na lista, sair da lista e usar a lista em
http://www.mat.puc-rio.br/~nicolau/olimp/obm-l.html
=


RE: [obm-l] Problema do Rei

2005-11-07 Por tôpico Qwert Smith
Acho que vc traduziu o problema errado.  Na versao em italiano que eu vi 
cada um via os chapeus de TODOS a sua frente.  O que tb e o caso em uma 
versao mais antiga do problema envolvendo apenas 2 cores.  Se de fato cada 
um so pode ver um chapel entao o numero minimo de sobreviventes sera 50.  Se 
da pra ver mais que um chapeu salvam-se muito mais magos.  Acho que da pra 
salvar 98, mas nao testei todos os casos ainda.




From: Carlos Eduardo Pereira [EMAIL PROTECTED]
Reply-To: obm-l@mat.puc-rio.br
To: Grupo OBM obm-l@mat.puc-rio.br
Subject: [obm-l] Problema do Rei
Date: Mon, 7 Nov 2005 12:12:42 -0200

Pessoal,

encontrei esse problema e estou tentando resolvê-lo para finalizar um
trabalho, se alguém tiver alguma maneira de resolvê-lo, serei muito
grato.

A cada ano na cidade de Wizardtown o rei convoca os seus 100 magos para uma
reunião que transcorre da seguinte forma: O rei coloca os magos em fila
indiana e põe um chapéu sobre a cabeça de cada um. O chapéu pode ser verde,
amarelo ou vermelho e cada mago pode ver somente o chapéu daquele que está 
a
sua frente. No final de cada minuto pelo menos um mago deve dizer uma cor 
e,

se mais de um mago quiser falar, deverão fazê-lo simultaneamente. Quem já
falou uma vez, deve ficar quieto até o final da reunião e quando todos
falarem, o rei fará decapitar aquele que tenha falado uma cor diferente
daquela de seu próprio chapéu.
Supondo que os magos tenham conhecimento de como ocorrerá a reunião e que
adotem uma estratégia que permita o maior número possível de acertos, para
salvarem-se, quantos magos sairão vivos? Qual será a estratégia adotada?

=
Instruções para entrar na lista, sair da lista e usar a lista em
http://www.mat.puc-rio.br/~nicolau/olimp/obm-l.html
=



=
Instruções para entrar na lista, sair da lista e usar a lista em
http://www.mat.puc-rio.br/~nicolau/olimp/obm-l.html
=


RE: [obm-l] PROBLEMAS DAT�VEIS!

2005-11-05 Por tôpico Qwert Smith

Vamos ver...


From: Jorge Luis Rodrigues e Silva Luis [EMAIL PROTECTED]

Turma! Tenho notado que, dentre os assuntos enviados à lista, esse é o 
único que ninguém simpatiza, pois nunca houve nenhum comentário e nem mesmo 
um protesto! Esquisito, não!


Houve mais quintas-feiras que terças-feiras no primeiro de dois anos 
consecutivos. Determine que dia da semana ocorreu com mais frequência no 
segundo ano, sabendo que nenhum desses dois anos foi bissexto.


ano = 365 = 7*52 +1
se teve mais quinta, quinta foi o 1o e ultimo dia do ano e no ano seguinte 
teve mais sexta




Em um certo ano houve exatamente 4 sextas-feiras e exatamente 4 
segundas-feiras em Janeiro. Em que dia da semana caiu o dia 20 de Janeiro 
nesse ano?


31 = 4*7 + 3

Como sexta e segunda nao estao entre os ultimos 3 eles tem que ser 3a, 4a e 
5a nessa ordem
Logo o primeiro dia do mes foi uma 3a e o 21o foi uma 2a... 20o dia foi um 
domingo




Os dias x de março e 2x de abril caem no mesmo dia da semana. Determine 
x. (bis!)


31 - x + 2x -1 = -1 (mod 7)
30 + x = -1 (mod 7)
2 + x = -1 (mod 7)
x=4 or x= 11




A propósito, se o mês de dezembro tem 4 domingos, em que dias da semana o 
dia de Natal não cai? (bis!)




De novo 31 = 4*7 + 3

Os ultimos 3 podem ser:
2a, 3a e 4a
3a, 4a e 5a
4a, 5a, e 6a
5a, 6a e sabado

dizendo de outra maneira..o 29o dia nao pode ser nem 6a, nem sabado, nem 
domingo

O que resulta em 25o dia nao podendo ser 2a, 3a nem 4a


=
Instruções para entrar na lista, sair da lista e usar a lista em
http://www.mat.puc-rio.br/~nicolau/olimp/obm-l.html
=


RE: [obm-l] Medida Positiva e Interior Vazio

2005-10-12 Por tôpico Qwert Smith





From: claudio\.buffara [EMAIL PROTECTED]

No mais, alguém já descobriu por que um chicote estala quando é usado?

[]s,
Claudio.


Today's Question:

   Why does a whip make a crack noise?



The Answer:

   The crack occurs when the wave of motion traveling down a whip 
surpasses the speed of sound. The wave can move so quickly because a whip 
tapers from the handle to the tip. When a whip is snapped, the momentum from 
the motion at the handle is conserved, and consequently the speed increases 
as the diameter of the whip decreases. Thus the wave gathers speed as it 
continues down the length of the whip, and when its velocity exceeds the 
speed of sound it produces a small sonic boom — the distinctive crack.


   —The Editor

source: http://www.infoplease.com/askeds


=
Instruções para entrar na lista, sair da lista e usar a lista em
http://www.mat.puc-rio.br/~nicolau/olimp/obm-l.html
=


*OFF TOPIC*RE: [obm-l] O HOMEM DE 40 MILH�ES DE D�LARES!

2005-10-07 Por tôpico Qwert Smith
Se vc gosta de futebol americano, vc sabe que o Steve Young ja nao joga a 
muito tempo.  Pra falar a verdade os 49ers tb nao 'jogam' a muito tempo.  
Sao assim como o flamengo da liga americana.  Um time outrora respeitado que 
faz no maximo figuracao.  Isso quando nao da vexame mesmo.  Traduzir 
quaterback como zagueiro e a coisa mais extranha(estranha?) que eu ja li.




From: Jorge Luis Rodrigues e Silva Luis [EMAIL PROTECTED]
Reply-To: obm-l@mat.puc-rio.br
To: obm-l@mat.puc-rio.br
Subject: [obm-l] O HOMEM DE 40 MILHÕES DE DÓLARES!
Date: Fri, 07 Oct 2005 12:54:10 +

Se você gosta de futebol americano, provavelmente ouviu falar de Steve 
Young, o zagueiro MVP (Most Valuable Player, ou o jogador mais importante) 
dos San Francisco 49 ers. [...]



=
Instruções para entrar na lista, sair da lista e usar a lista em
http://www.mat.puc-rio.br/~nicolau/olimp/obm-l.html
=


RE: [obm-l] PELO SIM, PELO N�O!

2005-09-14 Por tôpico Qwert Smith

Sao necessarias pelo menos 2 perguntas.

Escolha um dos individuos e peca a ele que identifique os desonestos.
Logo em seguida faca o mesmo pedido ao mesmo individuo.

Se ele for o honsto suas respostas seram iguais e ele nao se acusa nunca.
Caso contrario suas respostas serao diferentes.  Basta entao vc se valer da 
resposta em que ele se acusa como um dos desonestos, ja que nessa resposta 
ele esta falando a verdade.



From: Jorge Luis Rodrigues e Silva Luis [EMAIL PROTECTED]
Reply-To: obm-l@mat.puc-rio.br
To: obm-l@mat.puc-rio.br
Subject: [obm-l] PELO SIM, PELO NÃO!
Date: Mon, 12 Sep 2005 12:39:54 +

...

Cinco pessoas estão em uma sala. Uma delas é um sujeito honesto, que sempre 
diz sempre a verdade. As outras quatro alternam uma mentira e uma verdade e 
podem começar por qualquer uma das duas. Todos sabem quem é o sujeito 
honesto, menos você. Qual é o número mínimo de perguntas necessário para 
descobrir o honesto?


NOTA: Achei muito simpático o termo nosso herói ...  Abraços!




=
Instruções para entrar na lista, sair da lista e usar a lista em
http://www.mat.puc-rio.br/~nicolau/olimp/obm-l.html
=


RE: [obm-l] Problemas de Congru�ncia

2005-09-12 Por tôpico Qwert Smith
Nao sou a melhor pessoa pra fazer esses exercicios, mas como quase ninguem 
se manifestou vamos la.  Quase todos os problemas se resumem em escolher 
numeros menores pra ir simplificando as contas.  O numero de passos e o 
tamanho desses numeros menores e ao gosto do fregues e depende da sua boa 
vontade de fazer contas ou da capacidade da calculadora a sua frente.



From: Adroaldo Munhoz [EMAIL PROTECTED]

1) Determine o algarismo das unidades de 3^100

A resposta ja foi dada pelo cleber


2) Determine o resto da divisão de 37^13 por 17

37 = 3 ( mod 17 ) == 37^13 (mod 17) = 3^13 (mod 17)
a) 3^13 = 3^5*3^5*3^3
b) 3^5 = 243 = 5 (mod 17)
c) 3^3 = 27 = 10 (mod 17)
3^13 (mod17) = 5*5*10 (mod 17) = 250 (mod 17) = 12 (mod 17)


3) Mostre que 2^83 – 1 é divisível por 167

2^9 = 512, 167*3 = 501 == 2^9 = 11 (mod 167)
2^83=2^81*2^2=(2^9)^9*4
2^83 (mod 167) = 11^9*4 (mod 167)
11^3=1331, 167*8=1336 == 11^3 = -5 (mod 167)
11^9*4 ( mod 167) = (-5)^3*4 (mod 167) = -500 (mod 167) = 1 (mod 167)
2^83 -1 (mod 167) = 1 -1 (mod 167) = 0 (mod 167)

4) A que número entre 0 e 6 é congruente módulo 7 o produto 
11.18.2322.13.19 ?

11 = 4 mod 7, 18 = 4 mod 7, 13 = -1 mod 7, 19 = -2 mod 7, 2322 = -2 mod 7
4*-2*4*-2*-1 = -1*-1*-1 = -1 = 6 (mod 7)

5) Fermat conjecturou que todo número da forma Fn = 2^2 + 1 é primo, e 
provou que isto é verdade para n = 0,1,2,3,4. Porém, a afirmação é falsa 
para n = 5 já que Euler provou que F_5 é divisível por 641. Mostre isto 
usando congruências.

A conjectura nao era bem essa.  Era pra ser Fn = 2^(2^n)+1
Para n=5 significa F_5 = 2^32 + 1
E pede pra mostrar que 2^32 + 1 e divisivel por 641 o que e analogo ao 
problema (2)

2^16 = 154 ( mod 641) = 2^32 = (2^16)^2 = 154^2 (mod 641)
154^2 = 23716 = -1 (mod 641) == 2^32 +1 = -1 +1 = 0 (mod 641)



6) Mostre que o quadrado de qualquer inteiro é côngruo a zero ou 1 módulo 4


0 (mod 4) = 0^2 = 0 (mod 4)
1 (mod 4) = 1^2 = 1 (mod 4)
2 (mod 4) = 2^2 = 0 (mod 4)
3 (mod 4) = 3^2 = 1 (mod 4)

7) Mostre que o quadrado de qualquer inteiro é côngruo a zero , 1 ou 4 (mod 
8)

Analogo ao exercicio (6)

8) Se 4 for o maior inteiro que puder ser armazenado em um (micromicro) 
computador, qual será o resultado armazenado como resultado de 3 + 4 se a 
soma módulo 5 for usada ?

Nao sei se entendi ... 3+4 = 7 = 2 (mod 5) ... sera isso?


=
Instruções para entrar na lista, sair da lista e usar a lista em
http://www.mat.puc-rio.br/~nicolau/olimp/obm-l.html
=


RE: [obm-l] Re: ainda sobre alian�as.

2005-09-08 Por tôpico Qwert Smith
Vou colocar uma variante... suponha que vc sabe que um dos aneis e na 
verdade mais pesado.  A diferenca de peso e tao pequena que as pesagem tem 
que ser feitas em uma balanca ultra precisa que so existe em um laboratorio 
da NASA.  Como no problema original, vc so tem grana pra comprar 3 pesagens. 
 Vc manda os aneis pra NASA e diz na primeira pesagem use tais e tais 
aneis, na segunda use otros tantos e por ai vai.   Vc precisa escolher as 
pesagens de forma que ao receber o resultado das pesagens vc seja capaz de 
identificar o anel defeituoso.   E ai? Da pra fazer?



From: Nicolau C. Saldanha [EMAIL PROTECTED]
Reply-To: obm-l@mat.puc-rio.br
To: filipe junqueira [EMAIL PROTECTED]
CC: [EMAIL PROTECTED]
Subject: [obm-l] Re: ainda sobre alianças.
Date: Thu, 8 Sep 2005 09:32:51 -0300

On Wed, Sep 07, 2005 at 03:45:42PM -0300, filipe junqueira wrote:
   Caros amigos da lista  e nicolau.
 Conforme falei anteriormente sobre o problema das alianças,e sobre minha
 possivel solução envio em anexo a mesma em anexo no formato .doc
 Se tiverem um tempinho deem uma lida na solução e porfavor me enviem
 algum comentario, se esta certa inconpleta ou totalmente errada.
 Muito obrigado galera.

 ps: ainda to esperando a resolução do problema dos pontos


Por favor não mande arquivos em formatos proprietários como *.doc
para a lista: use texto.

Quanto a sua solução, ela me parece correta. Ela é aliás bastante
parecida com as soluções que estão nos arquivos da lista: existe
muito pouco espaço para diferentes soluções neste problema.

[]s, N.


=
Instruções para entrar na lista, sair da lista e usar a lista em
http://www.mat.puc-rio.br/~nicolau/olimp/obm-l.html
=



=
Instruções para entrar na lista, sair da lista e usar a lista em
http://www.mat.puc-rio.br/~nicolau/olimp/obm-l.html
=


RE: [obm-l] RES: [obm-l] Problema das Alian�as...

2005-08-30 Por tôpico Qwert Smith
Como o prof. Nicolau ja falou esse assunto ja foi mais que discutido.  O 
problema e interessante...atente pro fato que nao se sabe se a alianca e 
mais pesada ou leve.  A primeira pesagem no caso apenas eliminaria 4.   
Tente um pouco mais e sigua os links que o prof. indicou.



From: David Cardoso [EMAIL PROTECTED]

Divida as 12 em 3 grupos de 4.
Compara dois grupos na balança.
Com isso, vc determina em qual dos 4 grupos a aliança está.
Pegue esse grupo que vc acabou de terminar, com 4 alianças,  compare duas
elas.
Caso tenha empatado, faça a 3a. pesagem com as 2 alianças restantes e
descubra qual é.

[]'s
David



=
Instruções para entrar na lista, sair da lista e usar a lista em
http://www.mat.puc-rio.br/~nicolau/olimp/obm-l.html
=


RE: ENC: [obm-l] Problema do casal de filhos

2005-08-23 Por tôpico Qwert Smith

From: Luiz Viola [EMAIL PROTECTED]

A solução do livro é tal como a abaixo, proposta por Thiago Kufner,
publicada na nossa lista.

Mas eu não consigo aceitar naturalmente...



Abraço

Viola



Digamos que H representa um filho homem e M uma filha mulher.
Como o casal teve dois filhos, as possibilidades são (na ordem mais
velha, mais nova):

H, H
H, M
M, H
M, M

Na primeira situação descrita no problema, sabemos que a criança mais
velha é um menino. Só podemos ter duas das quatro situações acima:

H, M
H, H

Ou seja, para a outra criança (a mais nova) ser um menino, só há uma
situação entre duas possíveis. Por isso que a probabilidade é 1/2.

Na segunda situação, só sabemos que uma das duas crianças é menino. Ou
seja, das quatro situações possíveis, estamos lidando com apenas três
(as que possuem no mínimo um H):

H, H
H, M
M, H


Aqui vou descordar um pouquinho...vou dizer que existem 4 possibilidades

1. Entrou na sala o filho mais vellho entre 2 meninos
2. Entrou na sala o filho mais novo entre 2 meninos
3. Entrou na sala o filho mais velho que tem uma irmazinha
4. Entrou na sala o filho mais novo que tem uma irmazona


probabilidade 2/4 = 1/2 .  E ai, como e que fica?


Assim, temos apenas 1 entre 3 possibilidades que satisfazem o enunciado.
Portanto, para a situação 2, a probabilidade é 1/3.

[]'s
Kufner




=
Instruções para entrar na lista, sair da lista e usar a lista em
http://www.mat.puc-rio.br/~nicolau/olimp/obm-l.html
=


RE: [obm-l] geo plana

2005-08-23 Por tôpico Qwert Smith
Vamos la...o problema fala em alteracoes no lado causando alteracoes na 
area.  Como relacionar lado e area?  lado*lado = area
Agora que vc ja sabe o que vai usar e so escrever exatamente o que esta 
dito.

(lado + 2)*(lado + 2) = area + 36

tudo junto:
x^2 = y
(x+2)^2 = y + 36
4x + 4 = 36
x=8



From: elton francisco ferreira [EMAIL PROTECTED]

olá pessoal, queria saber como armar esse tipo de
questão, pois já tentei armar de várias maneiras mais
a resposta não coincide com a do livro.
desde já agradeço!

Determine o lado de um quadrado, sabendo que se
aumentado o lado em 2 cm, a sua área aumenta em 36 cm.



=
Instruções para entrar na lista, sair da lista e usar a lista em
http://www.mat.puc-rio.br/~nicolau/olimp/obm-l.html
=


RE: [obm-l] OUTRO PROBLEMA EM ABERTO!(de JOrge p/ a lista)

2005-08-09 Por tôpico Qwert Smith
Acho que e um problema de interpretacao.   Existem 256 possiveis grupamentos 
de assistentes.  Cada assistente estara presente em 128 desses grupamentos.  
Supondo que a sala tivesse apenas 5 escrivaninhas.  Em 29 dos 128 grupos nos 
quais assisnte A1 esta presente vai faltar cadeira pra todo mundo.  Se A1 e 
sempre o ultimo a chegar e acaba sempre em pe entao 5 cadeiras so nao da.  
Se eles tem um trato de alternar aqueles que ficam em pe entao  pode ser que 
5 de mas nao fiz as contas.



From: Chicao Valadares [EMAIL PROTECTED]

 Olá Chicão e demais colegas da lista! Este é mais um
 problema que vem me
 tirando o sono, pois a resposta dada pelo colega
 Cláudio Buffara não
 coincide com a do livro (6 escrivanhinhas).

 O Departamento de Matemática tem 8 assistentes que
 cursam pós-graduação e
 ocupam a mesma sala de estudos. As probabilidades de
 cada assistente estudar
 em casa ou na sala de estudos são iguais. Quantas
 escrivaninhas precisam ser
 colocadas na sala para que cada assistente tenha uma
 escrivaninha disponível
 pelo menos, 90% do tempo?

 Abraços!



=
Instruções para entrar na lista, sair da lista e usar a lista em
http://www.mat.puc-rio.br/~nicolau/olimp/obm-l.html
=


[obm-l] Magica Matematica

2005-07-29 Por tôpico Qwert Smith
Vc acaba de chegar pra sua primeira aula de um curso introdutorio de 
matematica.  O professor e seu assistente convidam todos os alunos a 
participar de um truque de cartas.  O prof. sai da sala enquanto o 
assistente pede aos alunos que escolham 5 cartas de um baralho normal de 52 
cartas.
O assistente pega as cinco escolhidas e arruma elas na mesa do prof.  sendo 
4 com o valor a mostra e uma virada.  O prof entao retorna pra sala e ao 
bater o olho nas cartas em sua mesa diz o valor e naipe da carta virada pra 
baixo.  Os alunos apludem, cocam a cabeca, procuram marcas nas cartas ate 
que o prof diz:  Vcs vao se dividir em pares e teram que fazer o mesmo 
truque pra turma.  Vai valer 80% da sua nota.  E ai?  Vai correr e pedir 
transferencia pra outra turma?  Compra um livro de magica pra tentar 
garantir a nota?  Como fazer o truque?



=
Instruções para entrar na lista, sair da lista e usar a lista em
http://www.mat.puc-rio.br/~nicolau/olimp/obm-l.html
=


Re: [obm-l] Magica Matematica

2005-07-29 Por tôpico Qwert Smith

Muito interessante a sua solucao.  Eu abri a porta quando descrevi
o problema :).  No problema original como me foi passado o assistente
podia escolher qual carta esconder e a ordem de entregar as 4 restantes
oa magico/prof.  Ou seja so poderia usar a leitura horizontal como vc 
colocou.




From: Ricardo Bittencourt [EMAIL PROTECTED]
Reply-To: obm-l@mat.puc-rio.br
To: obm-l@mat.puc-rio.br
Subject: Re: [obm-l] Magica Matematica
Date: Fri, 29 Jul 2005 12:09:28 -0300

Qwert Smith wrote:
O assistente pega as cinco escolhidas e arruma elas na mesa do prof.  
sendo 4 com o valor a mostra e uma virada.  O prof entao retorna pra sala 
e ao bater o olho nas cartas em sua mesa diz o valor e naipe da carta 
virada pra baixo.  Os alunos apludem, cocam a cabeca, procuram marcas nas 
cartas ate que o prof diz:  Vcs vao se dividir em pares e teram que fazer 
o mesmo truque pra turma.  Vai valer 80% da sua nota.  E ai?  Vai correr 
e pedir transferencia pra outra turma?  Compra um livro de magica pra 
tentar garantir a nota?  Como fazer o truque?


	Assumindo que o assistente possa ver a carta, e que tenha liberdade de 
deixar as cartas na mesa do jeito que quiser, é simples.


	Você cria um código de antemão que numera os naipes. 0=copas, 1=espadas, 
2=ouros, 3=paus, por exemplo. Na hora de dispor na mesa, você coloca à 
esquerda da carta escondida número equivalente de cartas abertas. Com isso 
o professor sabe de imediato o naipe.


	Isso é o arranjo na horizontal. Na vertical, o assistente escolhe colocar 
uma carta na mesma linha da escondida, ou acima dela, e o código é que 
carta abaixo=0, carta acima=1, e ele codifica o valor da carta em binário. 
Com 4 cartas, ele faz de 0 a 15, suficiente para todos os valores.


	Por exemplo, um 5 de espadas ficaria como abaixo. Notação: A=carta aberta, 
F=carta fechada, use fonte de espaçamento fixo.


-- A   A
-- A F   A


Ricardo Bittencourt   http://www.mundobizarro.tk
[EMAIL PROTECTED]  kimitatino kitiwa subete CATS ga itadaita
-- União contra o forward - crie suas proprias piadas --
=
Instruções para entrar na lista, sair da lista e usar a lista em
http://www.mat.puc-rio.br/~nicolau/olimp/obm-l.html
=



=
Instruções para entrar na lista, sair da lista e usar a lista em
http://www.mat.puc-rio.br/~nicolau/olimp/obm-l.html
=


RE: [obm-l] RE: [obm-l] Um problema de racioc�nio l�gico

2005-07-19 Por tôpico Qwert Smith

Caro Paulo,

Ao ler a questao imaginei justamente o ponto de vista do motorista, ou seja,
como se estivesse dentro dos carros e isso em nada afeta o resultado.
O que nao pode, como vc ja corretamente mencionou, e mudar o referencial
entre uma afirmacao e outra.  Eu acho que e um pouco de exageiro exigir que
isso seja parte do enunciado.


From: Paulo Santa Rita [EMAIL PROTECTED]

[...]


Estou supondo que alguem olha os carros pela frente e escuta as 
afirmacoes : TODAS AS AFIRMACOES SAO RELATIVAS A ESTE REFERENCIAL. A 
afirmacao IV foi mal formulada, pois a direita da ferrari pode levar 
alguem a se imaginar dentro do carro e, neste caso, a direita da ferrari 
significa a esquerda de quem olha pela frente.


Todavia, a banca pode objetar dizendo : nao e razoavel mudar o 
referencial ... Mas e certo que ela nao escreveu ESTA QUESTAO com 
suficiente clareza. Sera que eles terao a grandeza de reconhecer isso e 
alterar o gabarito ?


Um Abraco
Paulo Santa Rita
3,0800,190705




=
Instruções para entrar na lista, sair da lista e usar a lista em
http://www.mat.puc-rio.br/~nicolau/olimp/obm-l.html
=


RE: RES: [obm-l] Oswald de Souza (off)

2005-06-23 Por tôpico Qwert Smith




From: Artur Costa Steiner [EMAIL PROTECTED]

[snip]

Nesta lista mesmo hah exemplos disto. O Claudio Buffara, que conhece
diversas areas da matematica, parece ser um entusiasta da Teoria dos
Numeros.

Artur



O Super Buffara e tambem um exemplo de que existem otimos 'Matematicos' que
nao cursaram faculdade de Matematica.  Grande Claudio: eu quero ser que nem 
vc quando eu crescer :).



=
Instruções para entrar na lista, sair da lista e usar a lista em
http://www.mat.puc-rio.br/~nicolau/olimp/obm-l.html
=


RE: [obm-l] Clausula PROLOG para numeros primos

2005-06-02 Por tôpico Qwert Smith

From: Demétrius [EMAIL PROTECTED]

[...]
Verificar se existe divisão exata de X por todos os
números maiores que 1 e menores que X. Se não existir
o número é primo! (OK??!?!??!!!?)

Alguém teria uma alguma outra sujestão?!?!?



Basta testar so os numeros ate sqrt(X).


A minha cláusula PROLOG meia boca é:

% Entrada de um dado número X e o divisor de
% verificação par N = 2
primo(X, N):- N  X, A is (X mod N), A\=0, N1 is N+1,
primo(X, N1).

DEVERIA Executar Assim:
?primo(9, 2).
no.
?primo(11, 2).
yes.

Mas algo está errado Estou testando o 9 pois ele é
impar é pequeno e não é primo!!
[...]


Vc nao diz oque te leva a creer que algo esta errado.
Vc colocou o output esperado, mas nao colocou o que de fato retorna.
Sera que ta retornando sempre no?

Eu nao sei proplog mas acho que o yes ou no da resposta depende de todas
as ramificacoes serem ou nao verdadeiras

tente adicionar a seguinte clausula no comeco:
primo(X,X):- true.

Assim o seu programa mesmo que ineficiente deve te dar a resposta esperada.

Faz:
?- trace
?- primo(9,2)
?-primo(11,2)
?- notrace

Se vc quer uma ajuda mais detalhada em debugging o seu programa


=
Instruções para entrar na lista, sair da lista e usar a lista em
http://www.mat.puc-rio.br/~nicolau/olimp/obm-l.html
=


RE: [obm-l] Clausula PROLOG para numeros primos

2005-06-02 Por tôpico Qwert Smith

Usando a clausula que vc escreveu no email original eu recebi:
?- primo(9,2).
No.
?- primo(11,2).
No.

Depois que eu adicionei a linha que ja te mandei passou a funcionar:
?- primo(9,2).
No.
?-primo(11,2).
Yes.
?- primo( 12345341, 2 )
Yes.

O output que vc descreveu nao bate com a sua clausula original.  Da
um trace no programa.  De qualquer forma vc pode tentar esse aqui
tb que ja esta optimizado pra usar so ate a raiz quadrada de N.  Ainda
esta longe de ser o melhor, ja que precisamos testar apenas os primos
= sqrt(X):

/* -- versao um pouco melhorada -- */

/* se chegou ate 1 e pq X e primo */
primo( X, 1 ) :- true.

/* equanto N  1, teste ate achar A = 0 */
primo(X, N) :-
N  1,
A is (X mod N),
A\=0,
N1 is N-1,
primo(X, N1).

/* nao tem pq usar 2 argumentos nao e mesmo?
   Para X, escolha o maior inteiro = sqrt(x),
   que sera o maior divisor possivel de X. */
primo(X) :-
A is floor( sqrt( X ) ),
primo( X, A ).


=
Instruções para entrar na lista, sair da lista e usar a lista em
http://www.mat.puc-rio.br/~nicolau/olimp/obm-l.html
=


Re: [obm-l] Peso dos cachorros

2005-06-02 Por tôpico Qwert Smith

Pode estar certo, mas e meio que no chute.
De onde vc tirou que B+C = 950?
Pq nao B+C = 930 ou B+C=940?
Nada garante que a ordem va ser A+B,A+C,A+D,A+E,B+C,...
com ABCDE.  Basta usar como exemplo A=1,B=2,C=5,D=10,E=50.

O que e sempre verdade e que em ordem vem:
A+B,A+C,?,...,?,C+E,D+E .  O resto vc tem que mostrar.


From: Murilo Rebouças Fernandes de Lima [EMAIL PROTECTED]
  (+)A+B=900
  (+)A+C=920
  (-)B+C=950

  2A = 870 = A=435
  B=465
  C=485
  D=495
  E=505

  Eh assim?
- Original Message -
From: [EMAIL PROTECTED]

Em uma loja de animais há cinco cachorrinhos. O dono pesou os animais 
colocando dois de cada vez na balança, em todas as combinações possíveis. 
Por exemplo: Tico e Teco, depois Tico e Tuco, depois Teco e Tuco, e assim 
por diante. Os valores obtidos após todas as pesagens foram:

900g - 920g - 930g - 940g - 950g - 960g - 970g - 980g - 1000g - 1010g

A massa (peso) dos cinco cachorrinhos é:




=
Instruções para entrar na lista, sair da lista e usar a lista em
http://www.mat.puc-rio.br/~nicolau/olimp/obm-l.html
=


RE: [obm-l] [obm-l] Quest�es da minha lista de C�lculo!

2005-06-01 Por tôpico Qwert Smith




From: Joáo Vitor [EMAIL PROTECTED]


2. Seja  f : R- R uma função derivável satisfazendo a seguinte condição:

   f '(x) = c f(x) para todo x pertencente aos reais

Sendo c uma cte. Se g(x) = e^(-cx) * f(x) varifique g = K(cte) é contante
e conclua  que f(x) = k*e^(cx).



g(x) = f(x)/h(x) onde h(x) =e^(cx)

g'(x) = [ f'(x)h(x) - f(x)h'(x) ]/h^2(x)
g'(x) = [c*f(x)*e^(cx) - f(x)*ln(e^c)*e^(cx) ]/h^2(x)
g'(x) = [c*f(x)*e^(cx) - f(x)*c*e^(cx)]/h^2(x) = 0/h^2(x) = 0
g'(x) = 0 = g(x) = k

k = f(x)/h(x) = f(x) = k*h(x) = k*e^(cx)


=
Instruções para entrar na lista, sair da lista e usar a lista em
http://www.mat.puc-rio.br/~nicolau/olimp/obm-l.html
=


Re: [obm-l] Problema 1 da XXV OBM - N�vel 1, fase 3

2005-04-29 Por tôpico Qwert Smith
Na verdade ele chega em C(2,9)
Acho que o raciocinio e mais ou menos assim:
temos 9 nove espacos e precisamos escolher 2 deles pra colocar sinais '+'
Por exemplo:
Se escolhecemos espacos 3 e 6
_ _ + _ _ + _ _ _ ficariamos entao com 223.
Se escolhecemos espacos 2 e 3
_ + + _ _ _ _ _ _ ficariamos com 106
Espero que tenha ficado claro.
From: Eduardo Wilner [EMAIL PROTECTED]
  Prezado João Carlos
  Poderia explicar melhor tua solução?
  Parece que vc. chega a C(7,9)! De onde?
  Porque os algarismos resultam como restos da divisão
por 7?
  Eu encontrei 42 ...!
  Abraço
  Wilner

=
Instruções para entrar na lista, sair da lista e usar a lista em
http://www.mat.puc-rio.br/~nicolau/olimp/obm-l.html
=


RE: [obm-l] Siglas em Latim

2005-04-28 Por tôpico Qwert Smith
QED - Quod Erat Demonstrandum (which was to be demonstrated)
QEF - Quod Erat Faciendum (which was to be done)
QEI - Quod Erat Inveniendum (which was to be found out)

From: Bruno Bonagura [EMAIL PROTECTED]
Reply-To: obm-l@mat.puc-rio.br
To: obm-l@mat.puc-rio.br
Subject: [obm-l] Siglas em Latim
Date: Thu, 28 Apr 2005 22:19:12 -0300
No final de demonstrações é costume usar a sigla Q.E.D. para Quod Erat
Demonstrandum. Estou vendo os livros de Galileu e Newton e no fim de muita
demonstrações eles usam o Q.E.D. mas em algumas é usado o Q.E.I., alguem
sabe oque significa ? Procurei no google mas não encontrei.
Obrigado
Bruno Bonagura
=
Instruções para entrar na lista, sair da lista e usar a lista em
http://www.mat.puc-rio.br/~nicolau/olimp/obm-l.html
=

=
Instruções para entrar na lista, sair da lista e usar a lista em
http://www.mat.puc-rio.br/~nicolau/olimp/obm-l.html
=


Re: [obm-l] + duvidas para Qwert Smith

2005-04-25 Por tôpico Qwert Smith

From: Gustavo [EMAIL PROTECTED]
1) no da fita a primeira equação( x+y=3)  ok , mas.a segunda ... y=6-3x   
??
cada 1 hora gasta gravando em SP, tira a chance de gravarmos 3 horas em EP.
logo y (horas usando EP e igual ao maximo em EP (6) menos tres veses o
tempo gasto em SP (x)
2)João folga a cada 20 dias e Maria a cada 12 dias. Numa  certa semana,João
folgou na segunda-feira e Maria na sexta-feira. A partir dessa sexta-feira
em  que Maria folgou, o número de dias decorridos até que eles folguem no
mesmo dia,pela segunda vez, será ?   ESTA SERIA UMA VERSÃO MAIS COMPLICADA,
SERÁ QUE VC TERIA ALGUMA MANEIRA TÃO BOA QUANTO A OUTRA !! recebi uma dica
de usar o teorema do resto chines mas tive dificuldade de encontrá-lo 
pela
internete.

O problema aqui e que a resposta do outro problema nao era tao boa assim.
Vou tentar concertar nesse.  A ideia do mmc continua valendo.  O mmc e o
periodo entre folgas conjuntas... no caso desse problema:
A cada 60 dias ( mmc(20,12) ) eles folgam juntos.
Se ja x o dia que eles folgaram juntos por ultimo.
folgas do joao
x,x+20,x+40,(x+60)
folgas da Maria
x,x+12,x+24,x+36,x+48,(x+60)
Se Joao e Maria folgam juntos hj, entao folgaram de novo na 3a folga de Joao 
e
na 5a folga de Maria.  No problema anterior a resposta saiu certo pq as PA 
enumeravam
justamente os dias comecando da 1a folga dos personagens apos seu ultimo 
encontro.
O que NAO e o caso nesse problema.  Ficou faltando definir em que ponto do 
ciclo
estavamos.

Eu sugiro fazer assim:
1) PA do Joao
-4,16,36,56,... razao 20
2) PA da Maria
0,12,24,36,... razao 12
Seja N o dia do proximo encontro
1) N=-4 + 20x
2)N=0 +12y
de 1) e 2) temos 3) 5x=3y+1
Sabemos que o ciclo pro joao tem 3 folgas e pra Maria tem 5 folgas
ou seja 0x=3 e 0y= 5, sabemos ainda que x e y sao inteiros.
Dai temos x=2 e y=3, e N=36
=
Instruções para entrar na lista, sair da lista e usar a lista em
http://www.mat.puc-rio.br/~nicolau/olimp/obm-l.html
=


RE: [obm-l] + duvidas

2005-04-20 Por tôpico Qwert Smith

01.  Fitas de vídeo podem gravar nas velocidades SP e EP durante 2 e 6 
horas
respectivamente. Pretende-se gravar um filme com 3 horas de duração usando  
SP
o maior tempo possível e depois mudando para EP. Durante quantos minutos 
deverá
ser usada a velocidade EP?

tempo gasto gravando em SP = x
tempo gasto gravando em EP = y
x+y=3
y=6-3x
resolvendo o sistema
x=1.5 e y =1.5 Logo a resposta e 90 minutos.
02.  Renato estará de folga na próxima terça e após a cada seis dias. 
Roberto estará de folga na   próxima quarta e após a cada sete dias. A 
partir de hoje, que é segunda, em quantos dias, pela primeira vez, estarão 
os dois de folga  simultaneamente?

1 - Dias em que Renato esta de folga contando de segunda:
1,7,13,... PA de razao 6
2 - Dias em que Roberto esta de folga contando de segunda:
2,9,16,... PA de razao 7
mdc(6,7)=1 logo mmc(6,7) = 6x7 e o numero procurado sera o 7o termo de P1 ou
o 6o termo de P2.
Usando a_n = a_1 + (n-1)r em qualquer uma chegamos em a_n = 37
=
Instruções para entrar na lista, sair da lista e usar a lista em
http://www.mat.puc-rio.br/~nicolau/olimp/obm-l.html
=


RE: [obm-l] aee media aritimetica

2005-04-15 Por tôpico Qwert Smith
From: RAfitcho [EMAIL PROTECTED]
Considere um grupo de 10 pessoas A,B,C,...,I,J entre as quais:
I - A, B e C têm, respectivamente, 16, 29 e 31 anos
II - H e J nasceram em 1971
III - D, E, F, G e I nasceram, nessa ordem, em anos consecutivos.
Sabe-se ainda que todos ja aniversariaram neste ano (1998) e que a média 
aritimética das idades de todo o grupo é 23. O ano que I nasceu foi:

(A+B+C+D+E+F+G+H+I+J)/10 = 23 =
16+29+31+D+E+F+G+27+I+27=230 =
D+E+F+G+I = 100 =
(I-4+I)5/2 = 100 =
5I = 110 = I=22
é simples mas meu resultado ta dando outro deve ser alguma coisa que estou 
errando no meio mas se puderem me ajudar ficarei grato

[]'s

=
Instruções para entrar na lista, sair da lista e usar a lista em
http://www.mat.puc-rio.br/~nicolau/olimp/obm-l.html
=


RE: [obm-l] incoveniente!

2005-04-13 Por tôpico Qwert Smith
O inconveniente e perguntar a mesma coisa sempre.
Essa pergunta ja esteve na lista antes e se vc se desse ao trabalho
de procurar nos arquivos nao teria perguntado.
Independente de se a pergunta e facil ou dificil, o minimo de cortezia e 
procurar e ver se ja foi abordado antes.  Deveria ser obrigatorio o 
comprovamento de conhecimentos basicos de etiqueta antes de admissao na 
lista.

From: Tio Cabri st [EMAIL PROTECTED]
Caros amigos da lista
Já que no último ano postei o exercicio abaixo 2 ou 3 vezes e ninguém
responde, gostaria que me dissessem qual seria a razão de nínguém 
responder:
é muito fácil? é chato? é difícil?

Considerando o espaço de tempo de 24 horas, quantas vezes os ponteiros
das horas e dos minutos de um relógio formam 90º .

=
Instruções para entrar na lista, sair da lista e usar a lista em
http://www.mat.puc-rio.br/~nicolau/olimp/obm-l.html
=


Re: [obm-l] incoveniente!

2005-04-13 Por tôpico Qwert Smith
Essa e uma das vezes que essa pergunta ja esteve nessa lista:
http://www.mat.puc-rio.br/~nicolau/olimp/obm-l.200305/msg00170.html
Precisei digitar menos de 20 caracteres para achar ela usando o google.
E a resposta so nao e a mesma se o angulo for pi ou 0.  Nesse caso a 
resposta seria 22.

From: Tio Cabri st [EMAIL PROTECTED]
Reply-To: obm-l@mat.puc-rio.br
To: obm-l@mat.puc-rio.br
Subject: Re: [obm-l] incoveniente!
Date: Wed, 13 Apr 2005 18:37:11 -0300
Você é que está sendo muito descortês, grosso, mal-educado, etc.
Se vc não sabe ler eu a intitulei com a palavra 'incoveniente',
pq eu sabia que estava sendo incoveniente. Onde está a a minha agressão?
Sua resposta não tem fundamento pq estou nessa lista há anos e não me 
lembro
de ver essa pergunta respondida na lista. Eu a coloquei há 1 ano e como eu
já dissera ninguém havia respondido. Por isso fiz a pergunta abaixo.
E por isso a reformulei.
Eu não fui grosso em momento algum, ser grosso é:  mandar vc pegar sua
etiqueta e...
E mais: agradeço a tentativa dos Senhores Daniel e Marcelo só que continuo
incoveniente,
gostaria muito que alguém pudesse me ajudar nesse exercício, até mesmo
vc(Qwert) se souber.
Abraços
Cara...Vc me deixou fora do sério com sua ignorância.
- Original Message -
From: Qwert Smith [EMAIL PROTECTED]
To: obm-l@mat.puc-rio.br
Sent: Wednesday, April 13, 2005 5:06 PM
Subject: RE: [obm-l] incoveniente!

 O inconveniente e perguntar a mesma coisa sempre.
 Essa pergunta ja esteve na lista antes e se vc se desse ao trabalho
 de procurar nos arquivos nao teria perguntado.

 Independente de se a pergunta e facil ou dificil, o minimo de cortezia e
 procurar e ver se ja foi abordado antes.  Deveria ser obrigatorio o
 comprovamento de conhecimentos basicos de etiqueta antes de admissao na
 lista.

 From: Tio Cabri st [EMAIL PROTECTED]
 
 Caros amigos da lista
 Já que no último ano postei o exercicio abaixo 2 ou 3 vezes e ninguém
 responde, gostaria que me dissessem qual seria a razão de nínguém
 responder:
 é muito fácil? é chato? é difícil?
 
 Considerando o espaço de tempo de 24 horas, quantas vezes os ponteiros
 das horas e dos minutos de um relógio formam 90º .

=
Instruções para entrar na lista, sair da lista e usar a lista em
http://www.mat.puc-rio.br/~nicolau/olimp/obm-l.html
=


RE: [obm-l]

2005-04-13 Por tôpico Qwert Smith
A area pintada e a area do quadrado maior (36) menos as partes em branco
as partes em branco sao 4x^2 + 4A onde a e a area de cada um dos triangulos 
brancos

Recapitulando:
16 = 36 - 4x^2 - 4A
Agora so falta escrever A em funcao de x
cada triangulo e um triangulo reto isosceles com h=6-2x
c * sqrt(2) = 6 - 2x = c = (6-2x)/sqrt(2) = A = c^2/2 = (6-2x)^2/4
16 = 36 -4x^2 -36 +24x - 4x^2
dai pra frente nao tem misterio
From: RAfitcho [EMAIL PROTECTED]
Reply-To: obm-l@mat.puc-rio.br
To: obm-l@mat.puc-rio.br
Subject: [obm-l] Date: Wed, 13 Apr 2005 22:42:49 -0300
Ola
alguem consegue fazer a questão 3
desta página 
http://www.vestibular.ufrj.br/concursosanteriores/1997/dia2/matematica2.html

ficarei muito grato porque esse problema esta me deixando neurótico...
rafael

=
Instruções para entrar na lista, sair da lista e usar a lista em
http://www.mat.puc-rio.br/~nicolau/olimp/obm-l.html
=


RE: [obm-l] Duvidas

2005-04-13 Por tôpico Qwert Smith
1) De cada 3 latas temos 2 + x onde x = energia gasta pra lata nao reciclada
5% de x = 1 = x = 20
cada 3 latas gastam 22 unidades
em 24 latas 176 unidades
2)O preco por metro foi de x/40 para x/30 que significa um aumento de x/120
x/120 = 33% de x/40 resposta b.
From: matduvidas48 [EMAIL PROTECTED]
Reply-To: obm-l@mat.puc-rio.br
To: obm-l obm-l@mat.puc-rio.br
Subject: [obm-l] Duvidas
Date: Wed, 13 Apr 2005 23:10:56 -0300
 v n   01.A reciclagem de latas de alumínio permite uma 
considerável economia de energia elétrica: a produção de cada lata recic  l 
   reciclada gasta apenas 5% da energia que seria necessária para produzir 
uma lata não-reciclada.Considere que, de atrês latas 
produzidas, uma não é obtida por reciclagem, e que a produção de cada lata 
reciclada consome 1 unidade  unidade  de energia. De acordo com essa 
proporção, o número de unidades de energia necessário para a produção de
  24 latas é igual a:

   02. Um fabricante de papel higiênico reduziu o comprimento 
dos rolos de 40 m para 30 m. No entanto o preço dos rolos de papel 
higiênico, para o consumidor, manteve-se constante.   Nesse caso, é CORRETO 
afirmar que, para oconsumidor, o preço do metro de papel 
higiênico teve um aumento :
   a) inferior a 25%.b) superior ou igual a 30%.   
 c) igual a 25%.d) superior a 25% e inferior a 30%.

Agradeço desde de já.
Ary Ary Queiroz
__
Acabe com aquelas janelinhas que pulam na sua tela.
AntiPop-up UOL - É grátis!
http://antipopup.uol.com.br/


=
Instruções para entrar na lista, sair da lista e usar a lista em
http://www.mat.puc-rio.br/~nicolau/olimp/obm-l.html
=


RE: [obm-l] Re: [obm-l] quest�o de geometria

2005-03-31 Por tôpico Qwert Smith
O enunciado deve estar errado sim. Garanto que vc quis dizer OB
onde vc escreveu CB.
Ai da pra responder e a resposta e 1.  Basta desenhar o quadrilatero ACBD
e mostrar usando semelhancas de triangulos que o angulos opostos do 
quadrilatero
somam 180o e portanto e um quadrilatero inscritivel.  O ponto equististante 
dos vertices sera justamente o centro do circulo que circunscreve o 
quadrilatero.

From: Brunno [EMAIL PROTECTED]
Andre essa é uma questao do colégio naval e o enunciado esta identico,
questao chata neh
um abraco
- Original Message -
From: Eduardo Wilner [EMAIL PROTECTED]
   Oi Bruno.
   Creio que vc. quis dizer dois segmentos de retas
(diferentes)...
   Ainda assim, as posições dos pontos A,B,C e D não
ficam definidas.
   Favor verificar e esclarecer.
  []'s
  Wilner
--- Brunno [EMAIL PROTECTED] wrote:
 Boa tarde
 Poderiam me ajudar nesta questão

 Dois segmentos de uma reta AB e CD  interceptam-se
 interiormente no ponto O.

 Sabe-se que as medidas de AO e CB  são
 respectivamente, 3cm e 4 cm

 e que as medidas de  CO e OD  são, respectivamente,
 2cm e 6cm.

 Qual o número de pontos do plano, determinado por AB
 e CD

 que eqüidistam dos pontos  A, B, C e D

 Obrigado

=
Instruções para entrar na lista, sair da lista e usar a lista em
http://www.mat.puc-rio.br/~nicolau/olimp/obm-l.html
=


RE: [obm-l] ++Duvidas

2005-03-31 Por tôpico Qwert Smith
10a + b = 3ab =[ dividindo por a, ja que a0]
10 + b/a = 3b = Como b e inteiro, b/a tem que ser inteiro
e 1=b/a=9 =  11=3b=19 = 4=b=6
como sao so 3 possibilidades basta testar e ver que A=15 e B=24
A/B=5/8
From: matduvidas48 [EMAIL PROTECTED]
.Sejam A e B dois números de dois algarismos cada um e AB.
Sabendo-se que cada um desses números é igual ao triplo do produto de seus 
algarismos, qual a razão A/B?

=
Instruções para entrar na lista, sair da lista e usar a lista em
http://www.mat.puc-rio.br/~nicolau/olimp/obm-l.html
=


RE: [obm-l] Primo ou composto??? (corre��o)

2005-03-31 Por tôpico Qwert Smith
From: Rhilbert Rivera [EMAIL PROTECTED]
Desculpe Qwert Smith ( mas, mesmo assim obrigado) me enganei na hora de 
escrever. Na realidade o problema é:
 Determine o menor valor positivo de n tal que  2.n^2 + p, seja um
número inteiro  composto, onde p é um número primo.

Como eu queria dizer, para n=p temos uma solução. Mas, existe solução para 
n menor que p? É aí que eu me atrapalho.
Que tal colocar p em evidencia?
2.n^2 + p = (2.n^2/p + 1)p que e composto desde que (2.n^2/p +1)1
2.n^2/p  0 = n  sqrt(p/2)
Vamos testar: Seja p=17 o menor valor de n pela formula acima seria 3
2.3^2 + 17 = 35 que e composto
Porem essa formula so funciona pra p5.
Pra p=2 o numero e sempre composto pra n0 logo n=1
Pra p=3 ou p=5, n=p e de fato a solucao.
Ainda esta muito estranho esse problema
P.S. Foi você Qwert que me escreveu uma vez solicitando livros 
disponibilizados LEGALMENTE e gratuitamente na internet, além daqueles que 
eu coloquei na lista?
Nao fui eu nao
=
Instruções para entrar na lista, sair da lista e usar a lista em
http://www.mat.puc-rio.br/~nicolau/olimp/obm-l.html
=


RE: [obm-l] Quest�o de PA

2005-03-30 Por tôpico Qwert Smith
Faltou definir se 0 pertence a N ou nao.
Se vc inclui o zero a resposta e n^2 - n.
Se vc nao inclui o 0 a resposta e n^2 + n
Fica facil de testar escolhendo um n pequeno como 1 ou 2.
AAcho que o gabarito esta errado

From: Brunno [EMAIL PROTECTED]
Se P(x) x^a -1   é divisível por ( x+ 1) e a [pertence a] N, podemos 
afirmar que a soma dos n primeiros números a que satisfazem esta 
condição é

no gabarito indica n^2

=
Instruções para entrar na lista, sair da lista e usar a lista em
http://www.mat.puc-rio.br/~nicolau/olimp/obm-l.html
=


RE: [obm-l] Re: [obm-l] Re:[obm-l] Re: [obm-l] Quest�o de PA

2005-03-30 Por tôpico Qwert Smith
Vc esta confundindo as coisas.  P(x) = x^a - 1.  P(-1) = 0
Se  (x+1) divide P(x) entao a raiz de (x+1) tb e raiz de P(x)
-1 e raiz de (x+1) e consequentemente raiz de P(x)
Ser raiz significa que substituindo x por -1 o polinomio se anula.
em (x+1) = ( -1+1) = 0
em P(x) = P(-1) = 0 = -1^a -1 = 0
Melhorou?
From: Brunno [EMAIL PROTECTED]
Mas claudio pq P(x) tem que ser igual a zero???
  From: claudio.buffara
  x + 1 divide x^a - 1 == -1 é raiz de x^a - 1 == a é par, pois:
  (-1)^a - 1 = 0 se a é par e (-1)^a - 1 = -2 se a é ímpar
  Se 0 for natural, então os n primeiros valores de a são:
  0, 2, 4, ..., 2(n-1) == soma =  n(n-1)
  Caso contrário: 2, 4, 6, ..., 2n == soma = n(n+1)

=
Instruções para entrar na lista, sair da lista e usar a lista em
http://www.mat.puc-rio.br/~nicolau/olimp/obm-l.html
=


RE: [obm-l] Primo ou composto???

2005-03-30 Por tôpico Qwert Smith
n = 1
p.1^2 + p = 2p que e composto
From: Rhilbert Rivera [EMAIL PROTECTED]
Determine o menor valor positivo de n tal que  p.n^2 + p, seja um número  
composto, onde p é um número primo.

Comentários: É claro que para n = p o número é composto. O que estou me 
atrapalhando é como determinar se existe um n menor que p que satisfaça a 
condição do problema. Algo me diz que não existe esse n, ou ele não existe 
para alguns primos...

=
Instruções para entrar na lista, sair da lista e usar a lista em
http://www.mat.puc-rio.br/~nicolau/olimp/obm-l.html
=


Re: [obm-l] Principio das Gavetas

2005-03-29 Por tôpico Qwert Smith

From: Marcio M Rocha [EMAIL PROTECTED]
   Aproveitando a oportunidade, gostaria de uma sugestão no
problema
 seguinte: Prove que em qualquer seqüência de 39 números naturais
 consecutivos existe ao menos um número cuja soma dos algarismos é
 divisível por 11.

Esse parece interessante. Acho que vale a pena fazer umas simulações no 
Excel pra ver se você acha alguma periodicidade ou lei de formação. Se eu 
achar alguma coisa te falo.
 []s,
Claudio.

Seja N um numero terminado em 0 onde o algarismo das desenas nao e 9
Seja a = (soma dos algarismos de N) mod 11
A sequencia de 'mod 11's pelos proximos 9 numeros seria
a+1, a+2, a+3, a+4, a+5, a+6, a+7, a+8, a+9
Se a=0 o problema ja estaria resolvido, se a=2, nessa sequencia tb
teriamos um multiplo de 11, logo o pior caso e a=1
mas continuando a sequencia de 'mod 11's: o numero seguinte terminaria
em zero e seria a+1 (mod 11).  Os nove numeros da sequencia ja sabemos:
a+2, a+3, a+4, a+5, a+6, a+7, a+8, a+9, a+10.  Ou seja, em 19 numeros
em sequencia, nessas condicoes teremos certamente um multiplo de 11.
A questao e agora quantos numeros em sequencia sao necessarios pra
chegarmos em N?  E facil ver que na pior das hipoteses N seria o 20o de
uma sequencia de numeros naturais.  Nada impede que exista um multiplo
de 11 no meio, mas em 39 numeros teriamos obrigatoriamente no pior caso:
19 numeros quaisquer, N=a(mod11), e a sequencia de 19 numeros acima.
Acho que e isso nao?
=
Instruções para entrar na lista, sair da lista e usar a lista em
http://www.mat.puc-rio.br/~nicolau/olimp/obm-l.html
=


Re:[obm-l] Principio das Gavetas

2005-03-29 Por tôpico Qwert Smith
Vc comprovou a minha solucao anterior... o seu exemplo e justamente o worse 
case scenario:

3919 tem como soma de algarismos 22 que e divisivel por 11
From: claudio.buffara [EMAIL PROTECTED]
Reply-To: obm-l@mat.puc-rio.br
To: obm-l obm-l@mat.puc-rio.br
Subject: Re:[obm-l] Principio das Gavetas
Date: Tue, 29 Mar 2005 15:40:21 -0300
De:[EMAIL PROTECTED]
Para:obm-l@mat.puc-rio.br
Cópia:
Data:Tue, 29 Mar 2005 08:44:28 -0300
Assunto:[obm-l] Principio das Gavetas
 Aproveitando a oportunidade, gostaria de uma sugestão no problema
 seguinte: Prove que em qualquer seqüência de 39 números naturais
 consecutivos existe ao menos um número cuja soma dos algarismos é
 divisível por 11.

 []s,

 Márcio.

A afirmativa não é verdadeira.
Contra-exemplo:
3881, 3882, ..., 3919.
Por outro lado, acho que com 40 naturais consecutivos o resultado é 
verdadeiro.

Minha idéia foi considerar o termo da sequência que termina com o maior 
número possível de algarismos 9 (digamos k algarismos 9, com k = 1).
Chamando este termo de N e a soma de seus algarismos de S(N), eu descobri o 
contra-exemplo no caso em que S(N) == 10 e k == 6 (mod 11).

O seguinte lema (fácil de provar) foi útil:
Se N é um número natural que termina por k algarismos 9 (k = 0) e se S(N) 
é a soma dos algarismos de N, então S(N+1) = S(N) - 9k + 1.

[]s,
Claudio.

=
Instruções para entrar na lista, sair da lista e usar a lista em
http://www.mat.puc-rio.br/~nicolau/olimp/obm-l.html
=


Re: [obm-l] Principio das Gavetas

2005-03-29 Por tôpico Qwert Smith
remando minha solucao anterior no caso dela ter se perdido nas caixas 
postais virtuais da vida

From: Qwert Smith [EMAIL PROTECTED]
Reply-To: obm-l@mat.puc-rio.br
To: obm-l@mat.puc-rio.br
Subject: Re: [obm-l] Principio das Gavetas
Date: Tue, 29 Mar 2005 11:36:25 -0500

From: Marcio M Rocha [EMAIL PROTECTED]
   Aproveitando a oportunidade, gostaria de uma sugestão no
problema
 seguinte: Prove que em qualquer seqüência de 39 números naturais
 consecutivos existe ao menos um número cuja soma dos algarismos é
 divisível por 11.

Esse parece interessante. Acho que vale a pena fazer umas simulações no 
Excel pra ver se você acha alguma periodicidade ou lei de formação. Se eu 
achar alguma coisa te falo.
 []s,
Claudio.

Seja N um numero terminado em 0 onde o algarismo das desenas nao e 9
Seja a = (soma dos algarismos de N) mod 11
A sequencia de 'mod 11's pelos proximos 9 numeros seria
a+1, a+2, a+3, a+4, a+5, a+6, a+7, a+8, a+9
Se a=0 o problema ja estaria resolvido, se a=2, nessa sequencia tb
teriamos um multiplo de 11, logo o pior caso e a=1
mas continuando a sequencia de 'mod 11's: o numero seguinte terminaria
em zero e seria a+1 (mod 11).  Os nove numeros da sequencia ja sabemos:
a+2, a+3, a+4, a+5, a+6, a+7, a+8, a+9, a+10.  Ou seja, em 19 numeros
em sequencia, nessas condicoes teremos certamente um multiplo de 11.
A questao e agora quantos numeros em sequencia sao necessarios pra
chegarmos em N?  E facil ver que na pior das hipoteses N seria o 20o de
uma sequencia de numeros naturais.  Nada impede que exista um multiplo
de 11 no meio, mas em 39 numeros teriamos obrigatoriamente no pior caso:
19 numeros quaisquer, N=a(mod11), e a sequencia de 19 numeros acima.
Acho que e isso nao?
=
Instruções para entrar na lista, sair da lista e usar a lista em
http://www.mat.puc-rio.br/~nicolau/olimp/obm-l.html
=

=
Instruções para entrar na lista, sair da lista e usar a lista em
http://www.mat.puc-rio.br/~nicolau/olimp/obm-l.html
=


RE: [obm-l] Tres Probleminhas

2005-03-21 Por tôpico Qwert Smith
1) ta meio complicado... uma pergunta: vale usar raiz, mas ai o indice da 
raiz conta?

2) eu imagino que x-1-  seja x infinitesimamente menor que 1
vou rescrever a soma como
S = x + (x^4 - x^2 ) + ( x^16 - x^8 ) + (x^64 - x^32) + ... = x + S' onde
S' e a soma da PG com a1 = x^2(x^2-1) e q = x^4
logo S' = x^2(x^2-1)/(1-x^4) = (-1)x^2/(1+x^2) e
S = x - x^2/(1+x^2)
para x-1 acho que S - 1/2
3) ainda vou tentar
From: Claudio Buffara [EMAIL PROTECTED]
Pra quem nao tah fazendo nada neste fim de semana...
1. Expressar o numero 19 usando uma unica vez cada um dos numeros 1, 2 e 3 
e
mais as operacoes matematicas usuais (+, -, *, /, raizes, fatoriais, etc.).
Nao vale usar ponto decimal nem a funcao maior inteiro. (essa eh pro 
Qwert!)

2. Quanto vale lim(x - 1-) (x - x^2 + x^4 - x^8 + x^16 - x^32 + ...) ?
3. Sabe-se que a probabilidade de dois inteiros tomados ao acaso serem
primos entre si eh igual a 6/Pi^2. Tomando 4 inteiros a, b, c, d ao acaso 
(e
de forma independente) calcule a probabilidade de que mdc(a,b) = mdc(c,d).

[]s,
Claudio.

=
Instruções para entrar na lista, sair da lista e usar a lista em
http://www.mat.puc-rio.br/~nicolau/olimp/obm-l.html
=


RE: [obm-l] + ajuda

2005-03-14 Por tôpico Qwert Smith
Acho que seu gabarito esta todo errado, mas tudo bem...
From: Anna Luisa [EMAIL PROTECTED]
1) Um estudante estava praticando a sua aritmética adicionando os números 
das páginas do seu livro de matemática quando alguém o interrompeu. Ao 
retornar o exercício ele inadvertidamente incluiu o número de uma das 
páginas duas vezes na sua soma tendo encontrado 1986 como resultado final. 
O número dess página é:  (R: 34).
Para saber se esta certo o aluno faz a conta novamente usando a soma de uma 
PA

1986 = (1+ n)n/2 resolvendo a equacao de 2o grau ele acha x  0 discartado e 
x~=62.2
como o numero esperado era um inteiro ele sabe que teve erro.
a soma correta seria S=(1+62)62/2 = 1953 e ele contou 2 vezes a pagina 33

2) Um estudante em viagem de férias combinou com seu pai que se 
comunicariam em um código numérico no qual cada algarismo representaria uma 
letra distinta e como comprovação, o número representante da última palavra 
seria a soma dos anteriores. Sabendo que o estudante desejava enviar a 
mensagem SEND MORE MONEY podemos afirmar que a soma dos algarismos 
utilizados na mensagem codificada é igual a :  (R: 30)
SEND + MORE = MONEY = 9567 + 1085 = 10652  e a soma nao da 30 nao
3)Suprima cem dígitos do número 12345678910111213141516...5960 mantendo a 
ordem de modo a obter o menor número possível. A seguir, refaça o mesmo 
para obter o maior número possível. A soma dos algarismos desses dois 
números é:(R: 92)

Nao fiz ainda mas a julgar pelos anterios apostaria que nao e 92 nao.
=
Instruções para entrar na lista, sair da lista e usar a lista em
http://www.mat.puc-rio.br/~nicolau/olimp/obm-l.html
=


Re: [obm-l] Geometria II - A.C. Morgado, E. Wagner e M.Jorge - Duas questões conflitantes.

2005-03-14 Por tôpico Qwert Smith
No 33:
vc ja tinha chegado a conclusao que BD=x=10.
Do triiangulo ABD:
IDAI   AI 153
-- =  --   = --- = --- = ---
DB   AB   ID 102
letra B
=
Instruções para entrar na lista, sair da lista e usar a lista em
http://www.mat.puc-rio.br/~nicolau/olimp/obm-l.html
=


Re: [obm-l] Outros 2 do mesmo assunto...

2005-03-09 Por tôpico Qwert Smith
Na verdade, verdedadeira eu tava dando uma de Super Buffara e
deixei errado de proposito pra ver quem tava prestando atencao :).
Nao acreditou?  Ta bom... eu tinha feito 720+720 = 1480 :)
From: Bruno Bruno [EMAIL PROTECTED]
1- O navio 1 percorreu 720m, enquanto o navio 2 percorreu x. Depois, o
navio 1 percorreu x+400 e o navio 2 percorreu 720+720+x-400=1040+x.
720/x = x+400/x+1040
x^2+400x=720x+748800
x^2-320x-748800=0
x = 320+-1760/2
x'= 720 (nao serve)
x''= 1040
S=x+720 = 1760metros (não sei como o qwert chegou a 1900)

=
Instruções para entrar na lista, sair da lista e usar a lista em
http://www.mat.puc-rio.br/~nicolau/olimp/obm-l.html
=


RE: [obm-l] ex. fácil..

2005-03-09 Por tôpico Qwert Smith
Para a disatancia angular use
cos D = ( sin a )(sin b) + (cos a)(cos b)(cos P)
onde D = distancia angular entre A e B
   a = latitude do ponto A
   b = latitude do ponto B
   P = delta entre as longitudes de A e B
Depois do exercicio bracal vc acha que D ~= 3.16985
Usando a distancia media de 1 grau como 111 km,
chegamos a uma distancia de ~ 351.853 metros
Source: http://www.fes.uwaterloo.ca/crs/geog165/gcoords.htm
From: Vinícius Meireles Aleixo [EMAIL PROTECTED]
Olá
A latitude de SP é  23º33' S, 46º39' O  e do RJ é 22º53'S , 43º17' O. 
Calcule as distancias entre as duas cidades.

=
Instruções para entrar na lista, sair da lista e usar a lista em
http://www.mat.puc-rio.br/~nicolau/olimp/obm-l.html
=


Re: [obm-l] Equação

2005-03-09 Por tôpico Qwert Smith
Sera que nao faltou um parentesis no numerador?
(x^2 + 1)/(x+1)^2=3

From: Claudio Buffara [EMAIL PROTECTED]
Fazendo o obvio ==
x^4 + 2x^3 + x^2 + 1 = 3x^2 + 6x + 3 ==
x^4 + 2x^3 - 2x^2 - 6x - 2 = 0 ==
polinomio irredutivel por Eisenstein com p = 2 ==
nao vejo nenhuma solucao bonitinha
Essa equacao tem uma unica raiz real, igual a aproximadamente 1,6917395.
[]s,
Claudio.
on 09.03.05 18:35, Davidson Lima at [EMAIL PROTECTED] wrote:
  Meus amigos, como faço para resolver a questão:
  x^2+1/(x+1)^2=3
  Desde já agradeço a atenção.
  Davidson Estanislau

=
Instruções para entrar na lista, sair da lista e usar a lista em
http://www.mat.puc-rio.br/~nicolau/olimp/obm-l.html
=


Re: [obm-l] Equação

2005-03-09 Por tôpico Qwert Smith
Calma, desculpe!  Don't shoot the messenger
Eu nao sei se era pra ser com parentesis, mas se fosse ainda assim seria
uma questao dificil comparada com umas que tem aparecido esses dias :(.
From: Claudio Buffara [EMAIL PROTECTED]
Nesse caso eu nao sei o que eh mais deprimente numa lista que trata de
olimpiadas de matematica: alguem escrever x^2+1/(x+1)^2 quando queria dizer
(x^2+1)/(x+1)^2 ou alguem nao saber resolver uma misera equacao do 2o. 
grau.


=
Instruções para entrar na lista, sair da lista e usar a lista em
http://www.mat.puc-rio.br/~nicolau/olimp/obm-l.html
=


RE: [obm-l] Outros 2 do mesmo assunto...

2005-03-08 Por tôpico Qwert Smith
O 1o e uma simples regra de 3 e a resposta e 1900m
O 2o e uma dificilima equacao de PRIMEIRO grau e a resposta e 83
From: Alan Pellejero [EMAIL PROTECTED]
1) Dois barcos partem num mesmo instante de lados
opostos de um rio de margens paralelas. Viajam cada
qual perpendicularmente às margens, com velocidade
constante. Suponha que um deles é mais rápido que o
outro, e que se cruzam num ponto situado a 720m da
margem mais próxima. Completada a travessia, cada
barco fica parado no respectivo cais por 10 minutos.
Na volta eles se cruzam a 400m da outra margem. Qual a
largura do rio?
2)Um capitão quer colocar os seus soldados em filas,
formando um quadrado. Tendo colodado um certo número
de soldados em cada fila, sobraram 39 soldados;
colocando mais um soldado em cada fila, ficaram
faltando 50 soldados para completar o quadrado. O
número de soldados do batalhão é...
O 1 parece que está sem dados suficientes
Obrigado a todos pela ajuda!
Alan

=
Instruções para entrar na lista, sair da lista e usar a lista em
http://www.mat.puc-rio.br/~nicolau/olimp/obm-l.html
=


RE: [obm-l] Outros 2 do mesmo assunto...

2005-03-08 Por tôpico Qwert Smith
Heh... o segundo e dificil mesmo tem que saber elevar ao quadrado
a resposta certa e 1975
From: Qwert Smith [EMAIL PROTECTED]
O 1o e uma simples regra de 3 e a resposta e 1900m
O 2o e uma dificilima equacao de PRIMEIRO grau e a resposta e 83
From: Alan Pellejero [EMAIL PROTECTED]
1) Dois barcos partem num mesmo instante de lados
opostos de um rio de margens paralelas. Viajam cada
qual perpendicularmente às margens, com velocidade
constante. Suponha que um deles é mais rápido que o
outro, e que se cruzam num ponto situado a 720m da
margem mais próxima. Completada a travessia, cada
barco fica parado no respectivo cais por 10 minutos.
Na volta eles se cruzam a 400m da outra margem. Qual a
largura do rio?
2)Um capitão quer colocar os seus soldados em filas,
formando um quadrado. Tendo colodado um certo número
de soldados em cada fila, sobraram 39 soldados;
colocando mais um soldado em cada fila, ficaram
faltando 50 soldados para completar o quadrado. O
número de soldados do batalhão é...
O 1 parece que está sem dados suficientes
Obrigado a todos pela ajuda!
Alan

=
Instruções para entrar na lista, sair da lista e usar a lista em
http://www.mat.puc-rio.br/~nicolau/olimp/obm-l.html
=

=
Instruções para entrar na lista, sair da lista e usar a lista em
http://www.mat.puc-rio.br/~nicolau/olimp/obm-l.html
=


Re: [obm-l] questao de regra de tres composta

2005-03-04 Por tôpico Qwert Smith
Esta correto?
Vamos supor que nada mudou... ou seja a jornada de trabalho continua a mesma 
e o numero de trabalhadores tb.  Como 2/5 do trablho nessas condicoes levou 
10 dias, precisariamos de mais 15 dias pra terminar o servico.  Voce acha 
mesmo que diminuir os trabalhores e a carga horaria faz o trabalho acabar 
mais rapido?

From: Brunno [EMAIL PROTECTED]
Reply-To: obm-l@mat.puc-rio.br
To: obm-l@mat.puc-rio.br
Subject: Re: [obm-l] questao de regra de tres composta
Date: Fri, 4 Mar 2005 13:37:41 -0300
Esta correto, mas poderia mostrar como chegou a essa resposta
um abraco
- Original Message -
From: elton francisco ferreira [EMAIL PROTECTED]
To: obm-l@mat.puc-rio.br
Sent: Friday, March 04, 2005 11:17 AM
Subject: Re: [obm-l] questao de regra de tres composta
14 dias. se estiver errado, favor informar.
--- [EMAIL PROTECTED] wrote:
 Ola pessoal poderiam me ajudar nesta questao
  Se 2/5 de um trabalho foram feitos em 10 dias por
 24 operários que
  trabalhavam 7 horas por dia, então quantos dias
 serão necessários para
  terminar o trabalho, sabendo que 4 operários foram
 dispensados e que os
  restantes agora trabalham 6 horas por dia?
 
  Um abraco


 Conheça o novo iBest Acelerado e aumente a
 velocidade da sua navegação em até 5 vezes. O
 primeiro mês é gratuito. Basta acessar o endereço
 http://www.ibest.com.br/acelerado para se cadastrar.

=
 Instruções para entrar na lista, sair da lista e
 usar a lista em
 http://www.mat.puc-rio.br/~nicolau/olimp/obm-l.html

=



___
Yahoo! Acesso Grátis - Instale o discador do Yahoo! agora.
http://br.acesso.yahoo.com/ - Internet rápida e grátis
=
Instruções para entrar na lista, sair da lista e usar a lista em
http://www.mat.puc-rio.br/~nicolau/olimp/obm-l.html
=
=
Instruções para entrar na lista, sair da lista e usar a lista em
http://www.mat.puc-rio.br/~nicolau/olimp/obm-l.html
=

=
Instruções para entrar na lista, sair da lista e usar a lista em
http://www.mat.puc-rio.br/~nicolau/olimp/obm-l.html
=


RE: [obm-l] Derivadas parciais...

2005-03-03 Por tôpico Qwert Smith
d(log(x+y))/dx = d(log(x+y))/dy = 1/(x+y)?
From: [EMAIL PROTECTED]
Não tenho tempo de procurar...se alguém souber como se acham as derivadas 
de
f(x,y)=log(x+y )...alguém me pediu pra hoje e eu não lembro

=
Instruções para entrar na lista, sair da lista e usar a lista em
http://www.mat.puc-rio.br/~nicolau/olimp/obm-l.html
=


Re: [obm-l] RE: [obm-l] Exercício Conjuntos ITA! e UFRJ

2005-03-02 Por tôpico Qwert Smith
1- De I e III vem que 7% sao mulheres com mais de 30
2- De II vem que 32% sao mulheres
3- De IV e VII vem que 20% sao solteiras
4- De V e VI vem que 2% sao solteiras com mais de 30
a- De 1 e 2 vem que 25% sao mulheres = 30 anos
b- De 3 e 4 vem que 18% sao solteiras =30 anos
De a e b temos que 7% sao casadas = 30 anos
E isso que vc chama de enorme?
Certamente o problema e puramente bracal e, a meu ver, preconceituoso.
Implica que todo individuo que nao e solteiro e casado.
Implica que todo individuo que nao e homem e mulher :).
From: Alan Pellejero [EMAIL PROTECTED]
[...]
Olá amigos!
O 2 eu fiz, mas queria saber se há alguma maneira mais
eficiente...minha resolução ficou enorme! Um abraço!
Alan
[...]
 2-)Tendo sido feito um levantamento estatístico dos
 resultados do Censo Populacional 96 em uma cidade,
 descobriu-se, sobre a população que:
 I) 44% têm idade superior a 30 anos;
 II)68% são homens;
 III)37% são homens com mais de 30 anos;
 IV) 25% são homens solteiros;
 V) 4% são homens solteiros com mais de 30 anos;
 VI)6% são indivíduos solteiros com mais de 30 anos;
 VII) 45% dos indivíduos são solteiros.
 Com base nesses dados, pode-se afirmar que a % da
 população desta cidade que representa as mulheres
 casadas com idade igual ou inferior a 30 anos é de:

=
Instruções para entrar na lista, sair da lista e usar a lista em
http://www.mat.puc-rio.br/~nicolau/olimp/obm-l.html
=


RE: [obm-l] séries

2005-03-02 Por tôpico Qwert Smith
Acho que vc escreveu a serie errado.
Se a serie e:
numerador: 1, 2, 1, 1, 2, 1, 1, 2, 1 ...
denominador: 1, 3, 5, 7, 9, 11, 13 ...
sinais: +, +, +, -, -, -, +, +, + ...
vc pulou o 19 no denominador e a serie deveria ser na verdade
S[n]= 1 +2/3 +1/5 -1/7 -2/9 -1/11 +1/13 +2/15 +1/17
   -1/19 -2/21 -1/23 +1/25 +2/27 ...
Da pra dividir em 6 somas distintas, todas elas de inversos de PAs de razao 
12.

a) 1*(1+1/13+1/25+1/37+...)
b) 2*(1/3+1/15+1/27+...)
c) 1*(1/5+1/17+1/29+...)
d) -1*(1/7+1/19+1/31+...)
e)-2*(1/9+1/21+1/33+...)
f)-1*(1/11+1/23+1/35+...)
O Paulo mandou, faz pouco tempo, um email sobre series de inversos de PA e 
ai acabou.


From: Demetrio Freitas [EMAIL PROTECTED]
Saudações,
Um de séries, facilzinho para esquentar:
Calcule o valor para onde converge a soma:
S[n]= 1 +2/3 +1/5 -1/7 -2/9 -1/11 +1/13 +2/15 +1/17
-1/21 -2/23 -1/25 +1/27 +2/29 ...
Isto é:
numerador- 1 2 1 1 2 1 1 2 1 1...
sinais   - + + + - - - + + + -...
[]´s
Demétrio

=
Instruções para entrar na lista, sair da lista e usar a lista em
http://www.mat.puc-rio.br/~nicolau/olimp/obm-l.html
=


RE: [obm-l] Conicas, Triangulos e Divisores

2005-02-28 Por tôpico Qwert Smith
Eu acho que a resposta e 24... vou tentar explicar
Para n=25, n = 1*2*3*4*5*x ou escrevendo de outra forma 5#* 2*x
[# = primorial e * = multiplicacao]
Usando a prova de que existem infinitos primos *acho* que da pra mostrar que
p_n# * 2 e sempre maior p_(n+1)^2, logo se p_n e o maior primo = sqrt(n) 
entao
n e pelo menos p_n#*2 que e maior que p_(n+1)^2,
Logo n e pelo menos p_(n+1)#*2 que e maior que p_(n+2)^2, e nunca acaba

From: Claudio Buffara [EMAIL PROTECTED]
No mais, aqui vai um bonitinho de divisibilidade:
Qual o maior inteiro que eh divisivel por todos os inteiros positivos
menores do que sua raiz quadrada?

=
Instruções para entrar na lista, sair da lista e usar a lista em
http://www.mat.puc-rio.br/~nicolau/olimp/obm-l.html
=


RE: [obm-l] Questões de Lista de Conjuntos, C.Númericos e Lógica

2005-02-25 Por tôpico Qwert Smith
From: André Barreto [EMAIL PROTECTED]
[yada yada yada]
*** 9- Numa pesquisa entre os alunos do Elite foi constatado que 80% gostam 
de salada, 95% gostam de carne bovina, 10% gostam de peixe, 90% não gostam 
de frango e 15% não gostam de massas. Pergunta-se quantos alunos no mínimo 
não gostam de saladas, nem de carne bovina, nem de massas mas gostam de 
frango e peixe.
a) 10% b)15% c)20% d)25% e)30%

Essa questao deve estar errada, ja que nao existe nada no enunciado que 
garanta a existencia de alunos que gostem ao mesmo tempo de peixe e frango.  
Tambem vem do enunciado que somente 5% nao gostam de carne.  Logo o minimo e 
0% e o maximo e 5%.

=
Instruções para entrar na lista, sair da lista e usar a lista em
http://www.mat.puc-rio.br/~nicolau/olimp/obm-l.html
=


RE: [obm-l] Torneiras

2005-02-24 Por tôpico Qwert Smith
Se a torneira 1 enche o tanque em 15 horas, em 5 horas ela encheu 1/3 do 
tanque.
Logo resta a torneira 2 encher o restante (2/3).  Como a torneira 2 enche o 
tanque em 18, ela enche 2/3 em 12.  Como 12 - 5 = 7, ainda faltam 7 horas. 
Resposta (a).  Nao e bem um problema olimpico nao?
-Auggy

From: [EMAIL PROTECTED]
Um tanque tem duas torneiras.A primeira enche o tanque em 15 horas e a 
segunda em 18 horas.Estando o tanque vazio e abrindo-se as duas torneiras 
durante as primeiras  5 horas, enche-se uma parte do tanque.Podemos afirmar 
que , a segunda torneira encherá o restante do tanque em:
a) 7 horas
b) 8 horas
c) 13 horas
d) 10 horas
e) 8,5 horas

Agradeço desde de já

=
Instruções para entrar na lista, sair da lista e usar a lista em
http://www.mat.puc-rio.br/~nicolau/olimp/obm-l.html
=


Re: [obm-l] Moedas em sacos

2005-02-15 Por tôpico Qwert Smith
Acho ki vc pode usar mais informacao que te foi passada no enunciado.
1 - tem 10 moedas por saco.
2 - A balanca te da a diferenca entre os pratos
A informacao 2 acima e bastante relevante ja que com uma pesagem podemos 
saber exatamente quanto a mais cada moeda pesa.

Seguindo um raciocinio parecido com o seu cheguei a 126 sacos de moedas 
possiveis.
Vou deixar vc refazer depois ponho o raciocinio aqui.

From: Fernando Aires [EMAIL PROTECTED]
Reply-To: obm-l@mat.puc-rio.br
To: obm-l@mat.puc-rio.br
Subject: Re: [obm-l] Moedas em sacos
Date: Tue, 15 Feb 2005 13:05:28 -0200
Olá,
   Não sei se meu raciocínio está correto, mas eu pensei em resolver o
problema da seguinte forma:
   Como sabemos que o saco é mais pesado, para a última medição
(terceira), no pior caso, devemos ter 3 sacos. Mediríamos dois deles
na balança, e se um for mais pesado, é este; se ambos forem iguais, o
terceiro é o saco mais pesado.
   Dito isso, na segunda (penúltima) medição, devemos medir grupos de
3 sacos. Podemos medir 3 grupos, usando a mesma lógica da última
medição. Portanto, deve chegar 9 sacos na segunda medição.
   Assim, na primeira medição, pelo mesmo raciocínio, teremos 3 grupos
de 9 sacos. Portanto, o N máximo é 27.
   Espero que esteja certo...
Beijos,
--
--
Fernando Aires
[EMAIL PROTECTED]
Em tudo Amar e Servir
--
On Sat, 12 Feb 2005 10:57:42 -0200, Rogerio Ponce
[EMAIL PROTECTED] wrote:
 Ola' pessoal,

 Existem N sacos abertos com 10 moedas cada um.
 Um deles, defeituoso, tem 10 moedas iguais entre si, porem mais pesadas 
que
 o padrao. Os outros sacos tem as 10 moedas com o peso padrao (a 
principio
 desconhecido).

 Voce dispoe de uma balanca de 2 pratos, que fornece a diferenca de peso
 entre os pratos (prato da esquerda menos prato da direita).

 Qual o maior N que ainda permite a determinacao do saco defeituoso com
 apenas 3 leituras ?

 []'s
 Rogerio Ponce

=
Instruções para entrar na lista, sair da lista e usar a lista em
http://www.mat.puc-rio.br/~nicolau/olimp/obm-l.html
=

=
Instruções para entrar na lista, sair da lista e usar a lista em
http://www.mat.puc-rio.br/~nicolau/olimp/obm-l.html
=


Re: [obm-l] Moedas em sacos

2005-02-15 Por tôpico Qwert Smith
Exatamente.  E 126 tb e muito pouco...agora to achando que o maximo e 171.
Daqui a pouco mudo de ideia denovo
From: Rogerio Ponce [EMAIL PROTECTED]
Reply-To: obm-l@mat.puc-rio.br
To: obm-l@mat.puc-rio.br
Subject: Re: [obm-l] Moedas em sacos
Date: Tue, 15 Feb 2005 13:06:26 -0300
Ola' Fernando,
N=27 ainda e' pouco.
Repare que vc esta' apenas usando a informacao de um dos pratos pesar mais 
que o outro, sem considerar o valor dessa diferenca, fornecido pela 
balanca.
O fato e' que N pode ser mais alto que 27.

[]'s
Rogerio Ponce

From: Fernando Aires
Olá,
   Não sei se meu raciocínio está correto, mas eu pensei em resolver o
problema da seguinte forma:
   Como sabemos que o saco é mais pesado, para a última medição
(terceira), no pior caso, devemos ter 3 sacos. Mediríamos dois deles
na balança, e se um for mais pesado, é este; se ambos forem iguais, o
terceiro é o saco mais pesado.
   Dito isso, na segunda (penúltima) medição, devemos medir grupos de
3 sacos. Podemos medir 3 grupos, usando a mesma lógica da última
medição. Portanto, deve chegar 9 sacos na segunda medição.
   Assim, na primeira medição, pelo mesmo raciocínio, teremos 3 grupos
de 9 sacos. Portanto, o N máximo é 27.
   Espero que esteja certo...
On Sat, 12 Feb 2005 10:57:42 -0200, Rogerio Ponce wrote:
 Ola' pessoal,

 Existem N sacos abertos com 10 moedas cada um.
 Um deles, defeituoso, tem 10 moedas iguais entre si, porem mais pesadas 
que
 o padrao. Os outros sacos tem as 10 moedas com o peso padrao (a 
principio
 desconhecido).

 Voce dispoe de uma balanca de 2 pratos, que fornece a diferenca de peso
 entre os pratos (prato da esquerda menos prato da direita).

 Qual o maior N que ainda permite a determinacao do saco defeituoso com
 apenas 3 leituras ?

 []'s
 Rogerio Ponce
_
MSN Messenger: converse online com seus amigos .  
http://messenger.msn.com.br

=
Instruções para entrar na lista, sair da lista e usar a lista em
http://www.mat.puc-rio.br/~nicolau/olimp/obm-l.html
=

=
Instruções para entrar na lista, sair da lista e usar a lista em
http://www.mat.puc-rio.br/~nicolau/olimp/obm-l.html
=


Re: [obm-l] Moedas em sacos

2005-02-15 Por tôpico Qwert Smith
Legal, eu tinha limitado a ultima pesagem a 21 sacos fazendo
Prato 1:
1 moeda do saco 1
2 moedas do saco 2
3 moedas do saco 3
...
10 moeadas do saco 10
Prato 2:
1 moeda do saco 11
2 moedas do saco 12
3 moedas do saco 13
...
10 moedas do saco 20
saco 21 ficando de fora da pesagem
Certamente menos eficiente que seu metodo porem usava o fato que nao 
precisamos pesar todos os sacos.

De cara ja da pra aumentar N de 242 pra 287
Faz exatamente oque vc fez e deixa 45 sacos de fora... se os pratos nao 
acusarem diferenca na primeira pesagem usamos meu metodo pouco eficiente de 
pesar 21 sacos de cada lado com 3 sobrando.  Dai ou a terceira pesagem 
resolve entre os 21 sacos do prato mais pesado ou entre os 3 sacos que nunca 
foram pra balanca.  Certamente usando o seu metodo mais eficiente vai dar 
pra deixar de fora bem mais que 45 na primeira pesagem, mas ainda nao fiz as 
contas.  A questao e...como provar que o seu metodo e de fato o mais 
eficiente?

From: claudio.buffara [EMAIL PROTECTED]
Reply-To: obm-l@mat.puc-rio.br
To: obm-l obm-l@mat.puc-rio.br
Subject: Re: [obm-l] Moedas em sacos
Date: Tue, 15 Feb 2005 14:57:21 -0300
On Sat, 12 Feb 2005 10:57:42 -0200, Rogerio Ponce
wrote:
 Ola' pessoal,

 Existem N sacos abertos com 10 moedas cada um.
 Um deles, defeituoso, tem 10 moedas iguais entre si, porem mais pesadas 
que
 o padrao. Os outros sacos tem as 10 moedas com o peso padrao (a 
principio
 desconhecido).

 Voce dispoe de uma balanca de 2 pratos, que fornece a diferenca de peso
 entre os pratos (prato da esquerda menos prato da direita).

 Qual o maior N que ainda permite a determinacao do saco defeituoso com
 apenas 3 leituras ?

 []'s
 Rogerio Ponce


Eu achei N = 242 mas não sei provar que este é o maior N possível.
Suponhamos que uma moeda normal pese P e uma moeda mais pesada pese P+Q.
PRIMEIRA PESAGEM:
Colocamos 121 sacos num prato e 121 no outro.
A balança indicará em que prato está o saco mais pesado e o também o valor 
de Q, igual a 1/10 da leitura da balança.

SEGUNDA PESAGEM:
Numeramos os 121 sacos que incluem o mais pesado de 0 a 120 e fazemos o 
seguinte:
Sacos 0 a 10: 0 moedas no prato da E e 10 moedas no prato da D;
Sacos 11 a 21: 1 moeda no prato da E e 9 moedas no prato da D;
...
Sacos 11k a 11k+10: k moedas no prato da E e 10-k moedas no prato da D;
...
Sacos 110 a 120: 10 moedas no prato da E e 0 moedas no prato da D.

(obs: estou supondo que mesmo após colocar as moedas nos pratos da balança, 
continuamos a saber de que saco elas vieram. Por exemplo, podemos empilhar 
as moedas de um mesmo saco e operar a balança com cuidado de forma que as 
pilhas não desabem)

Suponhamos que o número do saco mais pesado seja 11k + r (0 = r = 10).
Nesse caso, os pesos em cada prato serão:
E = 605P + kQ
e
D = 605P + (10-k)Q
Logo, leitura da balança = E - D = (2k-10)Q.
Como já sabemos o valor de Q, ficaremos sabendo o valor de k.
Ou seja, após esta segunda pesagem, ficaremos sabendo que o saco mais 
pesado é um dos 11 seguintes: 11k, 11k+1, ..., 11k+10.

TERCEIRA PESAGEM:
Re-numeramos os 11 sacos que incluem o mais pesado de 0 a 10 e fazemos o 
seguinte:
Saco 0: 0 moedas no prato da E e 10 moedas no prato da D;
Saco 1: 1 moeda no prato da E e 9 moedas no prato da D;
...
Saco m: m moedas no prato da E e 10-m moedas no prato da D;
...
Saco 10: 10 moedas no prato da E e 0 moedas no prato da D.

Suponhamos que o saco mais pesado seja o m-ésimo.
Os pesos em cada prato serão:
E = 55P + mQ
e
D = 55P + (10-m)Q.
Leitura da balança = E - D = (2m-10)Q.
Como conhecemos Q, podemos determinar m e acabou.
[]s,
Claudio.

=
Instruções para entrar na lista, sair da lista e usar a lista em
http://www.mat.puc-rio.br/~nicolau/olimp/obm-l.html
=


Re: [obm-l] Moedas em sacos

2005-02-15 Por tôpico Qwert Smith
Ok N=927 and counting...
veja o meu reply pro email do Claudio
From: Bruno Bruno [EMAIL PROTECTED]
Reply-To: obm-l@mat.puc-rio.br
To: obm-l@mat.puc-rio.br
Subject: Re: [obm-l] Moedas em sacos
Date: Tue, 15 Feb 2005 18:30:04 -0300
Claudio, inspirado no seu raciocínio consegui chegar a 883.
(Desculpe o plágio, mas gostei da sua idéia)
Suponhamos que uma moeda normal pese P e uma moeda mais pesada pese P+Q.
1a pesagem:
Colocamos 441 sacos num prato e 441 no outro. Se ficarem iguais
obviamente será o outro saco, mas como isso é quase impossivel,
a balança indicará em que prato está o saco mais pesado e o também o
valor de Q, igual a 1/10 da leitura da balança.
2a pesagem:
Numeramos os 441 sacos de 0 a 440 e fazemos o seguinte:
Sacos de 0 a 20  --- nao pesamos.
Sacos de 21 a 41 --- uma moeda na esquerda
Sacos de 42 a 62 --- duas moedas na esquerda
...
Sacos de 21k a 21k+20 --- k moedas na esquerda
...
Sacos de 210 a 230 --- 10 moedas na esquerda
Repetindo o processo com os outros sacos, para o lado direito da
balança, chegamos finalmente ate o saco 440.
Pesando, já conhecido o valor de Q, chegamos a um grupo final de 21
sacos, dentre os quais estará o mais pesado.
Terceira pesagem:
Renumerando os sacos de 0 a 20, faremos o seguinte:
Não pesaremos o saco 0.
Colocaremos K moedas do saco K na esquerda (caso 0K11)
e colocaremos K moedas do saco K + 10 na direita (0K11)
Assim, concluiremos qual o saco mais pesado.
Por que acho (não tenho a menor certeza) que esse é o maior numero 
possivel:
A ultima pesagem pode ter no máximo 21 sacos. A 1a pesagem deve ser
usada para descobrirmos o valor de Q, fundamental para as proximas
pesagens. Assim, a 2a pesagem deve reduzir de (N-1)/2 para 21. Como
são 10 moedas em cada saco, podemos fazer 21 grupos, o que faz com que
(N-1)/2max = 21^2 = 441  = N=883

ps.: tentando rapidamente generalizar o problema, caso existam X
moedas em cada saco, Nmax = [(2X+1)^2]*2 + 1.  Ou caso existam X
moedas, e Y pesagens, Nmax = [(2X+1)^(Y-1)]*2+1
Ou ainda no remoto caso de haverem Z pratos (imaginem só, uma balança
de prato com 3, 8, 20 pratos... ainda bem que isso é matematica, nao
fisica) Nmax = [(ZX+1)^(Y-1)]*Z + 1. Acho q me empolguei, desculpem.

On Tue, 15 Feb 2005 15:59:25 -0300, Rogerio Ponce
[EMAIL PROTECTED] wrote:
 Caro Claudio,
 como sempre a sua engenhosidade é bem vinda.
 Mas N pode ser ainda maior...
 Grande abraço,
 Rogério.

 From: claudio.buffara  Ola' pessoal,
  
   Existem N sacos abertos com 10 moedas cada um.
   Um deles, defeituoso, tem 10 moedas iguais entre si, porem mais 
pesadas
 que
   o padrao. Os outros sacos tem as 10 moedas com o peso padrao (a
 principio
   desconhecido).
  
   Voce dispoe de uma balanca de 2 pratos, que fornece a diferenca de 
peso
   entre os pratos (prato da esquerda menos prato da direita).
  
   Qual o maior N que ainda permite a determinacao do saco defeituoso 
com
   apenas 3 leituras ?
  
   []'s
   Rogerio Ponce
  
 
 Eu achei N = 242 mas não sei provar que este é o maior N possível.
 
 Suponhamos que uma moeda normal pese P e uma moeda mais pesada pese 
P+Q.
 
 PRIMEIRA PESAGEM:
 Colocamos 121 sacos num prato e 121 no outro.
 A balança indicará em que prato está o saco mais pesado e o também o 
valor
 de Q, igual a 1/10 da leitura da balança.
 
 SEGUNDA PESAGEM:
 Numeramos os 121 sacos que incluem o mais pesado de 0 a 120 e fazemos o
 seguinte:
 Sacos 0 a 10: 0 moedas no prato da E e 10 moedas no prato da D;
 Sacos 11 a 21: 1 moeda no prato da E e 9 moedas no prato da D;
 ...
 Sacos 11k a 11k+10: k moedas no prato da E e 10-k moedas no prato da D;
 ...
 Sacos 110 a 120: 10 moedas no prato da E e 0 moedas no prato da D.
 
 (obs: estou supondo que mesmo após colocar as moedas nos pratos da 
balança,
 continuamos a saber de que saco elas vieram. Por exemplo, podemos 
empilhar
 as moedas de um mesmo saco e operar a balança com cuidado de forma que 
as
 pilhas não desabem)
 
 Suponhamos que o número do saco mais pesado seja 11k + r (0 = r = 
10).
 Nesse caso, os pesos em cada prato serão:
 E = 605P + kQ
 e
 D = 605P + (10-k)Q
 Logo, leitura da balança = E - D = (2k-10)Q.
 Como já sabemos o valor de Q, ficaremos sabendo o valor de k.
 
 Ou seja, após esta segunda pesagem, ficaremos sabendo que o saco mais
 pesado é um dos 11 seguintes: 11k, 11k+1, ..., 11k+10.
 
 TERCEIRA PESAGEM:
 Re-numeramos os 11 sacos que incluem o mais pesado de 0 a 10 e fazemos 
o
 seguinte:
 Saco 0: 0 moedas no prato da E e 10 moedas no prato da D;
 Saco 1: 1 moeda no prato da E e 9 moedas no prato da D;
 ...
 Saco m: m moedas no prato da E e 10-m moedas no prato da D;
 ...
 Saco 10: 10 moedas no prato da E e 0 moedas no prato da D.
 
 Suponhamos que o saco mais pesado seja o m-ésimo.
 Os pesos em cada prato serão:
 E = 55P + mQ
 e
 D = 55P + (10-m)Q.
 
 Leitura da balança = E - D = (2m-10)Q.
 Como conhecemos Q, podemos determinar m e acabou.
 
 []s,
 Claudio.

 _
 Chegou o que faltava: MSN Acesso 

RE: [obm-l] Associação OBM

2005-02-02 Por tôpico Qwert Smith
Associados fora do Brasil tb podem receber a Eureka?
From: Olimpiada Brasileira de Matematica [EMAIL PROTECTED]
Reply-To: obm-l@mat.puc-rio.br
To: obm-l@mat.puc-rio.br
Subject: [obm-l] Associação OBM
Date: Tue, 01 Feb 2005 10:54:18 -0200
Caros Olímpicos:
O endereço eletrônico da página da Associação Olimpíada
Brasileira de Matemática é:
http://www.obm.org.br/frameset-associacao.htm
Abraços, Nelly.
=
Instruções para entrar na lista, sair da lista e usar a lista em
http://www.mat.puc-rio.br/~nicolau/olimp/obm-l.html
=

=
Instruções para entrar na lista, sair da lista e usar a lista em
http://www.mat.puc-rio.br/~nicolau/olimp/obm-l.html
=


RE: [obm-l] PROBLEMinha

2005-01-16 Por tôpico Qwert Smith
seja
From: Fabio [EMAIL PROTECTED]
Uma turma com 100 alunos fez um teste com duas questões. Verificou-se na 
correção que:
1.1) 30 alunos acertaram apenas uma questão;
1.2) entre os que acertaram a primeira questão, 2/3 também acertaram a 
segunda questão;
1.3) entre os que erraram a primeira questão, 3/4 também erraram a segunda 
questão;
Determine quantos alunos erraram as duas questões
a) 15 b) 18 c) 24 d) 28   e) 30

Seja a  os alunos que acertaram somente a primeira questao
Seja b os alunos que acertaram somente a segunda questao
1.1) a + b = 30
1.2) 2a = alunos que acertaram as 2 questoes
1.3) 3b = alunos que erraram as 2 questoes
juntando 1.2, 1.3 e o enunciado
3a + 4b  = 100
a  +  b  =   30
Logo b=10, e 30 alunos erram as duas questoes.
=
Instruções para entrar na lista, sair da lista e usar a lista em
http://www.mat.puc-rio.br/~nicolau/olimp/obm-l.html
=


RE: [obm-l] Trapezio e Paralelogramo

2004-12-15 Por tôpico Qwert Smith

From: Claudio Buffara [EMAIL PROTECTED]
Reply-To: [EMAIL PROTECTED]
To: Lista OBM [EMAIL PROTECTED]
Subject: [obm-l] Trapezio e Paralelogramo
Date: Wed, 15 Dec 2004 11:44:43 -0200
Questoes de definicao:
1) Um trapezio tem exatamente um par de lados opostos paralelos ou pode ter
ambos os pares de lados opostos paralelos?
http://www.mail-archive.com/obm-l@mat.puc-rio.br/msg19411.html
2) Se o segundo caso for verdade, posso dizer que um paralelogramo eh um
trapezio isosceles?
http://www.m-w.com/cgi-bin/dictionary?book=Dictionaryva=Isoscelesx=9y=11
[]s,
Claudio.
=
Instruções para entrar na lista, sair da lista e usar a lista em
http://www.mat.puc-rio.br/~nicolau/olimp/obm-l.html
=

=
Instruções para entrar na lista, sair da lista e usar a lista em
http://www.mat.puc-rio.br/~nicolau/olimp/obm-l.html
=


Re: [obm-l] Trapezio e Paralelogramo

2004-12-15 Por tôpico Qwert Smith
So complementando...
From: Qwert Smith [EMAIL PROTECTED]
From: Claudio Buffara [EMAIL PROTECTED]
Por outro lado, segundo o Merriam-Webster, um trapezio isosceles tem os
lados nao-paralelos iguais. Isso implica que um trapezio precisa ter um 
par
de lados nao-paralelos, o que nao ocorre com um paralelogramo.
A inferencai esta errada. O dicionario diz (certo ou errado) que
para um trapezio ser isosceles precisa ter um par de lados nao paralelos.
Nao diz que todo trapezio precisa ter lado nao paralelos.
.
O que vc falou nao e necessariamente verdade... um paralelogramo nao tem
lados opostos nao-paralelos, mas um quadrado por exemplo tem 2 lados iguais 
e
nao paralelos.  Pelo dicionario o quadrado seria um trapezio isosceles e um 
retangulo
nao seria.  Eu acho que faltou eles dizerem lados OPOSTOS nao paralelos.
faltou dizer retangulo nao quadrado, ja que todo quadrado e retangulo :)
Em suma, suas duas referencias sao contraditorias e minha duvida
permanece...
Eu nao sou capaz de tirar minhas proprias duvidas,
que dira as duvidas de outro :), mas discordo que
sejam referencias contraditorias.
[]s,
Claudio.

=
Instruções para entrar na lista, sair da lista e usar a lista em
http://www.mat.puc-rio.br/~nicolau/olimp/obm-l.html
=


[obm-l] fatorando RSA

2004-12-07 Por tôpico Qwert Smith
A algum tempo atraz tinha um maluco (adjetivo carinhoso) aqui na lista que 
dizia que tinha revolucionado a fatoracao de numeros RSA.  Estou chamando de 
numeros RSA o produto de 2 primos grandes.  Na epoca me interessei pelo 
problema e agora que o trabalho diminuiu de ritmo volto a me interessar. 
Tendo em consideracao que o numero de fatores eh sempre 2 me parece que 
Quadratic Sieve seja o metodo mais logico de fatoracao, mas ociosidade, ou 
mesmo falta de juizo me fazem pensar que achar um metodo mirabolante e 
rapido seja possivel.  Me parece que a maior parte do tempo eh gasto 
tentando determinar se um dos possiveis fatores eh de fato primo.  Como nao 
sei matematica mas estava com tempo de sobra resolvi procurar substituir o 
teste de primaridade por outro que na minha ignorancia parece mais simples e 
rapido.  Acabei com algo assim:

1) se N = pq entao p e q sao as solucoes da equacao x^2 - (p+q)x + N = 0.
2) De N(mod 10^t) da pra achar os valores possiveis de (p+q) (mod 10^t)
3) Consideracoes de um leigo:
3.1) o conjunto de valores possiveis em (2) e menor que o numero de primos 
em um  intervalo
de 10^t numeros.
3.2) testar se ((p+q)^2 - 4N) e quadrado perfeito eh mais rapido que testar 
a primaridade de um dos primos de (3.1)

Pergunta: Ja da pra eu ir buscar minha Fields ou devo manter meu day job por 
enquanto?

[],
Auggy
=
Instruções para entrar na lista, sair da lista e usar a lista em
http://www.mat.puc-rio.br/~nicolau/olimp/obm-l.html
=


RE: [obm-l] 2 Problemas de Potenciação

2004-11-28 Por tôpico Qwert Smith

Olá!!!
Meu nome é João Ricardo, sou professor do Ensino Médio e acabo de me 
inscrever nesta lista.
Gostaria de aproveitar a oportunidade para propor 2 problemas que considero 
interessantes; o primeiro deles, elaborei para os meus alunos, já o segundo 
(que não consegui encontrar resposta certa), foi um problema da CMO 2003 - 
Olimpíada Canadense de Matemática.
Aqui vão eles:

1) Qual o último dígito de 2004^2004?
2) Quais os três últimos dígitos de 2003^(2002^2001)?
Tentei resolver o segundo da mesma maneira que resolvi o primeiro, e não 
consegui encontrar o resultado correto.

Abraços!!!
João Ricardo
2003 = 3 (mod 1000) logo 2003^(2002^2001) (mod 1000) = 3^(2002^2001) (mod 
1000)
3^100 = 1 ( mod 1000 ) entao se 2002^2001 = 100*s + t, temos que
3^(2002^2001) = 3^(100*s + t ) = 3^100^s * 3^t = 1^s * 3^t = 3^t ( mod 1000 
)

Precisamos entao descobrir t ou 2002^2001 ( mod 100 ) que eh 52
2003^(2002^2001) ( mod 1000 ) = 3^52 ( mod 1000) = 241
=
Instruções para entrar na lista, sair da lista e usar a lista em
http://www.mat.puc-rio.br/~nicolau/olimp/obm-l.html
=


RE: [obm-l] Problemas de Lógica

2004-11-28 Por tôpico Qwert Smith

From: Joÿe3o Ricardo [EMAIL PROTECTED]
  Olá pessoal!!!
  Hoje, vi alguns problemas de lógica que, apesar de simples, são 
bastante interessantes:

1) Quantas flores tenho se todas elas são rosas exceto duas, todas são 
tulípas exceto duas, e todas elas são margaridas exceto duas?

r + t + m = r + 2 = t + 2 = m + 2 = 3

2) Em uma folha de papel estão escritas 100 afirmações:
  1. Exatamente uma afirmação nesta página é falsa.
   2. Exatamente duas afirmações nesta página são falsas.
   3. Exatamente três afirmações nesta página são falsas.
   4. Exatamente quatro afirmações nesta página são falsas.
   5. Exatamente cinco afirmações nesta página são falsas.
   ...
   99. Exatamente noventa e nove afirmações nesta página são falsas.
   100.Exatamente cem afirmações nesta página são falsas.
Quantas afirmações são falsas e quantas são verdadeiras, nesta página?
1 afirmacao verdadeira, a de numero 99
O que aconteceria se Exatamente fosse substituído por Pelo menos?
As 50 primeiras afirmacoes poderiam ser verdadeiras
3) Duas caixas fechadas A e B possuem, cada uma, uma etiqueta contendo uma 
afirmação. A etiqueta da caixa A diz A afirmação na etiqueta de B é 
verdadeira e o ouro está na caixa A. A etiqueta na caixa B diz A 
afirmação na etiqueta de A é falsa e o ouro está na caixa A.
Assumindo que uma das caixas contém ouro, em qual delas ele se encontra?

As 2 afirmacoes tem que ser falsas para que nao haja contradicao, logo o 
ouro tem que estar em B

Divirtam-se
Abraços
João Ricardo
-
Yahoo! Acesso Grátis - Internet rápida e grátis. Instale o discador agora!

=
Instruções para entrar na lista, sair da lista e usar a lista em
http://www.mat.puc-rio.br/~nicolau/olimp/obm-l.html
=


RE: [obm-l] RECREAÇÃO!

2004-11-27 Por tôpico Qwert Smith

From: eritotutor [EMAIL PROTECTED]
...
Amigo Qwert ele disse no minimo
Entao fica facil...poe todos os baloes no teto
e o minimo por parede eh zero :)
=
Instruções para entrar na lista, sair da lista e usar a lista em
http://www.mat.puc-rio.br/~nicolau/olimp/obm-l.html
=


RE: [obm-l] RECREAÇÃO!

2004-11-26 Por tôpico Qwert Smith
From: eritotutor [EMAIL PROTECTED]
(...)
 Um quarto quadrado deve ser enfeitado com 14
balões de tal maneira que, em cada
 parede fique igual número deles. Quantos balões
no mínimo vou colocar em cada
 lado?
Coloque 3 baloes em cada parede e dois no teto
Sao 4 baloes por parede... tire os dois baloes do teto
e coloque em cantos opostos de forma que cada balao
pertence a 2 paredes
=
Instruções para entrar na lista, sair da lista e usar a lista em
http://www.mat.puc-rio.br/~nicolau/olimp/obm-l.html
=


RE: [obm-l] primos

2004-11-11 Por tôpico Qwert Smith
Nao da pra fechar um pouco mais o problema nao?
Mesmo com k maximo = 6 esse problema parece
que pode dar um numero muito grande.  Nao sei se
do jeito que foi proposto pode ser escrito em funcao
de k.
seja f(k) o problema proposto
f(6) = 1, pq so existe um conjunto de 6 primos
consecutivos que o produto e  5.  Logo
g=z=(2*3*5*7*11*13) , e g - z = 0.
f(5) = 3.  g e z em {2310, 15015}, logo existem
3 possiveis (g-z)s.
Ate aqui parece facil, mas daqui pra baixo os valores
possiveis pra g e z crescem muito rapido.
para k=1, temos 5133 possiveis g e z.  E para varios
g e z distintos a diferenca (g-z) = 2 ou -2.   Nao sei se
da pra resolver isso na mao nao.  Vou ter apelar
e escrever um programinha e ainda assim parece que vai
rodar algumas horas antes de cuspir a resposta.  Alguem
mais tem uma opiniao a respeito?
From: eritotutor [EMAIL PROTECTED]
Reply-To: [EMAIL PROTECTED]
To: obm-l [EMAIL PROTECTED]
Subject: [obm-l] primos
Date: Wed, 10 Nov 2004 20:24:46 -0200
Boa noite amigos,
* O produto de k primos consecutivos eh menor que
5.
** A soma de k primos consecutivos eh menor que
5.
Seja p1, p2, ...pk tal que *  e ** sao
satisfeitas.
Sejam tb g1, g2, ...gk tal que *  e ** sao
satisfeitas.
Seja q = p1*p2*...*pk e z = Quantos (em funcao de k) numeros inteiros 
menores
que 5 podem ser expressos na forma  q - z .


=
Instruções para entrar na lista, sair da lista e usar a lista em
http://www.mat.puc-rio.br/~nicolau/olimp/obm-l.html
=


  1   2   3   >